Stack Exchange Network

Stack Exchange network consists of 183 Q&A communities including Stack Overflow , the largest, most trusted online community for developers to learn, share their knowledge, and build their careers.

Q&A for work

Connect and share knowledge within a single location that is structured and easy to search.

Is there a consensus in philosophy and other humanities that PhD dissertations have to be at least 200 pages long?

I've read a blog post written by Robert Wolff saying that for a philosophy dissertation to be considered good, at least in American universities, it should be between 200 to 250 pages long.

We could respond, of course, that it is not necessarily the number of pages or word count that makes a thesis up to par. If this is so, can you please cite a dissertation wherein a student was able to defend his central claim in less than a hundred pages?

Community's user avatar

  • 19 (1) Can you cite the blog post? (2) Is this individual's blog post actually going to impact what universities accept? In other words, why is this worth researching and discussing? (3) Why <100 when the post talks about 200-250? (4) Comment: the fact that someone was able to successfully defend a thesis in <100 pages doesn't mean it would meet some arbitrary criteria for being "good" according to Dr. Wolff (see "no true scotsman" fallacy ). –  eykanal Commented Mar 13, 2017 at 14:34
  • 4 I think that this quote applies s-media-cache-ak0.pinimg.com/736x/05/ce/44/… –  Matt Commented Mar 13, 2017 at 17:06
  • 2 The statement comes from a 2010 blog post named "How to write a doctoral dissertation in philosophy" : "An American dissertation in Philosophy should be about 200 to 250 pages long, and have five or six chapters." Note that Wolff is talking about Philosophy specifically, not about other humanities as it is indicated in the question title. In fact, he explicitly contrasts a Philosophy dissertation with those from Anthropology or History. –  Schmuddi Commented Mar 13, 2017 at 22:03
  • 1 "If this is so, can you please site a dissertation wherein a student was able to defend his central claim in less than a hundred pages." Some purely technical ways to get there would be using smaller font sizes, smaller page margins, preferencing shorter synonymous words, getting rid of unnecessary particles, really small footnotes. More content oriented approaches could include not repeating yourself or scrutinizing content for if it is really necessary for the central claim. –  NoDataDumpNoContribution Commented Mar 14, 2017 at 8:44
  • 8 "A dissertation is either good or long." (German saying.) –  Daniel Commented Mar 14, 2017 at 11:01

3 Answers 3

The University of Minnesota library system maintaining electronic dissertations library contained 2,536 records for Phd students.

The range was incredibly variable (minimum of 21 pages, maximum of 2002), but most dissertations were around 100 to 200 pages.

enter image description here

Source: https://beckmw.wordpress.com/2013/04/15/how-long-is-the-average-dissertation/

In fact, it mostly depends on the research area, the university regulation and supervisor research 'style'.

Krebto's user avatar

  • 4 Some areas have more or less entropy in the way that information is transmitted. For example, law tend to be more verbose than maths... –  woliveirajr Commented Mar 13, 2017 at 20:07
  • 19 Unfortunately, the graph doesn't list Philosophy, which is what Wolff is talking about exclusively in his post. So, strictly speaking, the observation that the average page numbers vary substantially between disciplines doesn't really answer the question (unless we concede that the OP has misread the blog post by Wolff). –  Schmuddi Commented Mar 13, 2017 at 22:13
  • 11 2002 pages sure puts things into perspective –  mbrig Commented Mar 13, 2017 at 23:18
  • 16 @woliveirajr For maths it has been claimed that one sentence is enough! –  Ben C Commented Mar 14, 2017 at 9:36
  • 3 @mbrig The 2002 pages probably included large tables of data and/or source code of computer programs. –  David Richerby Commented Mar 14, 2017 at 13:13

The answer is no, there is no such consensus.

Page number is a very poor indicator of a document's quality unlike what some people seem to believe .

Don't worry about what a blogger says, talk to your adviser about her or his expectations in terms of the depth and length of your thesis, and how your current version stands with them.

Your adviser should know the standards for your institution and has a shared interest in your thesis meeting them.

Cape Code's user avatar

  • 3 -1 While your response seems like it ought to be true, I think it avoids answering the question. The asker is looking for proof of the existence of a high-quality short PhD thesis. Actually, they ask for a citation to one, which is a little more than just proof that it exists. –  Clumsy cat Commented Mar 14, 2017 at 12:19
  • @TheoreticalPerson this answer addresses the question in the title. There is no universal standard as to what constitute an acceptable thesis, let alone a good one. So asking for an example is pointless. –  Cape Code Commented Mar 14, 2017 at 13:23
  • 2 Some people just deal better with a concrete example of X, than an assertion that there are overwhelming reasons why X surely must exist. Where in this case X is, "a short thesis that defends its main claim". –  Steve Jessop Commented Mar 14, 2017 at 13:27
  • 1 @CapeCode I'm not sure your second point really follows from your first. There may be no standard for a thesis, but there are many interesting observations that might be made from a short thesis that some professor considered "good enough". –  Clumsy cat Commented Mar 14, 2017 at 13:42

The helpful chart supplied by @Krebto tells the story well: dissertations in the humanities are generally longer than those in the science and social sciences. The shortest humanities dissertations on the list are in English, at 220pp. History runs 295. In my experience, Philosophy is in between those two.

None of this says anything about intellectual quality, but it is a good indicator of expectations.

Philly's user avatar

You must log in to answer this question.

Not the answer you're looking for browse other questions tagged phd thesis humanities philosophy ..

  • Featured on Meta
  • Bringing clarity to status tag usage on meta sites
  • We've made changes to our Terms of Service & Privacy Policy - July 2024
  • Announcing a change to the data-dump process

Hot Network Questions

  • A "clearly" won queen endgame
  • Can You Build a Propeller or Airfoil for a Higgs Field?
  • GNU grep: This manpage is not compatible with mandoc
  • Font showing in Pages but not in Font book (or other apps)
  • How can I prove both series are equal?
  • Generating Carmichaeal numbers in polynomial time
  • Is there an efficient way to extract a slice of a 3d array?
  • Easyjet denied EU261 compensation for flight cancellation during Crowdstrike: Any escalation or other recourse?
  • Does the ship of Theseus have any impact on our perspective of life and death?
  • Very old fantasy adventure movie where the princess is captured by evil, for evil, and turned evil
  • Trying to identify manufacturer logo .. circle with circle inside
  • Is the oil level here too high that it needs to be drained or can I leave it?
  • How common is it for external contractors to manage internal teams, and how can we navigate this situation?
  • Creating a deadly "minimum altitude limit" in an airship setting
  • What's the airplane with the smallest ratio of wingspan to fuselage width?
  • Are the peer reviewers of a journal article allowed to voice surprise to the editor at a "minor revision" decision?
  • Definition of the electric field
  • John 1:1, "the Word was with God, and the Word was God." Vs2, He was in the beginning with God. Does this not prove Jesus existed before Genesis 1:1?
  • How Subjective is Entropy Really?
  • MOSFETs keep shorting way below rated current
  • Is magnetic flux in a transformer proportional to voltage or current?
  • Did the United States have consent from Texas to cede a piece of land that was part of Texas?
  • Optimal Algorithm to Append and Access Tree Data
  • Who is affected by Obscured areas?

how long is a humanities dissertation

offer

How to Write a Social Science or Humanities Thesis/Dissertation

how long is a humanities dissertation

Writing a thesis/dissertation is a huge task, and it is common to feel overwhelmed at the start. A thesis and a dissertation are both long pieces of focused research written as the sum of your graduate or postgraduate course.

The difference between a thesis and a dissertation can depend on which part of the world you are in. In Europe, a dissertation is written as part of a Master’s degree, while a thesis is written by doctoral students. In the US, a thesis is generally the major research paper written by Master’s students to complete their programs, while a dissertation is written at the doctoral level.

The purpose of both types of research is generally the same: to demonstrate that you, the student, is capable of performing a degree of original, structured, long-term research. Writing a thesis/dissertation gives you experience in project planning and management, and allows you the opportunity to develop your expertise in a particular subject of interest. In that sense, a thesis/dissertation is a luxury, as you are allowed time and resources to pursue your own personal academic interest.

Writing a thesis/dissertation is a larger project than the shorter papers you likely wrote in your coursework. Therefore, the structure of a thesis/dissertation can differ from what you are used to. It may also differ based on what field you are in and what kind of research you do. In this article, we’ll look at how to structure a humanities or social science thesis/dissertation and offer some tips for writing such a big paper. Once you have a solid understanding of how your thesis/dissertation should be structured, you will be ready to begin writing.

How are humanities and social science thesis/dissertations structured?

The structure of a thesis/dissertation will vary depending on the topic, your academic discipline, methodology, and the place you are studying in. Generally, social science and humanities theses/dissertations are structured differently from those in natural sciences, as there are differences in methodologies and sources. However, some social science theses/dissertations can use the same format as natural science dissertations, especially if it heavily uses quantitative research methods. Such theses/dissertations generally follow the “IMRAD” model :

  • Introduction

Social science theses/dissertations often range from 80-120 pages in length.

Humanities thesis/dissertations, on the other hand, are often structured more like long essays. This is because these theses/dissertations rely more heavily on discussions of previous literature and/or case studies. They build up an argument around a central thesis citing literature and case studies as examples. Humanities theses/dissertations tend to range from between 100-300 pages in length.

The parts of a dissertation: Starting out

Never assume what your reader knows! Explain every step of your process clearly and concisely as you write, and structure your thesis/dissertation with this goal in mind.

As you prepare your topic and structure your social science or humanities thesis/dissertation, always keep your audience in mind. Who are you writing for? Even if your topic is other experts in the field, you should aim to write in sufficient detail that someone unfamiliar with your topic could follow along. Never assume what your reader knows! Explain every step of your process clearly and concisely as you write, and structure your thesis/dissertation with this goal in mind.

While the structure of social science and humanities theses/dissertations differ somewhat, they both have some basic elements in common. Both types will typically begin with the following elements:

What is the title of your paper?

A good title is catchy and concisely indicates what your paper is about. This page also likely has your name, department and advisor information, and ID number. However, the specific information listed varies by institution.

Acknowledgments page

Many people probably helped you write your thesis/dissertation. If you want to say thank you, this is the place where it can be included.

Your abstract is a one-page summary (300 words or less) of your entire paper. Beginning with your thesis/dissertation question and a brief background information, it explains your research and findings. This is what most people will read before they decide whether to read your paper or not, so you should make it compelling and to the point.

Table of contents

This section lists the chapter and subchapter titles along with their page numbers. It should be written to help your reader easily navigate through your thesis/dissertation.

While these elements are found at the beginning of your humanities or social science thesis/dissertation, most people write them last. Otherwise, they’ll undergo a lot of needless revisions, particularly the table of contents, as you revise, edit, and proofread your thesis/dissertation.

The parts of a humanities thesis/dissertation

As we mentioned above, humanities and some social science theses/dissertations follow an essay-like structure . A typical humanities thesis/dissertation structure includes the following chapters:

  • References (Bibliography)

The number of themes above was merely chosen as an example.

In a humanities thesis/dissertation, the introduction and background are often not separate chapters. The introduction and background of a humanities thesis/dissertation introduces the overall topic and provides the reader with a guide for how you will approach the issue. You can then explain why the topic is of interest, highlight the main debates in the field, and provide background information. Then you explain what you are investigating and why. You should also specifically indicate your hypothesis before moving on to the first thematic chapter. 

Thematic chapters (and you can have as many of them as your thesis/dissertation guidelines allow) are generally structured as follows:

  • Introduction: Briefly introduce the theme of the chapter and inform the reader what you are going to talk about.
  • Argument : State the argument the chapter presents
  • Material : Discuss the material you will be using
  • Analysis : Provide an analysis of the materials used
  • Conclusion : How does this relate to your main argument and connect to the next theme chapter?

Finally, the conclusion of your paper will bring everything together and summarize your argument clearly. This is followed by the references or bibliography section, which lists all of the sources you cited in your thesis/dissertation.

The parts of a social science thesis/dissertation

In contrast to the essay structure of a humanities thesis/dissertation, a typical social science thesis/dissertation structure includes the following chapters:

  • Literature Review
  • Methodology

Unlike the humanities thesis/dissertation, the introduction and literature review sections are clearly separated in a social science thesis/dissertation. The introduction tells your reader what you will talk about and presents the significance of your topic within the broader context. By the end of your introduction, it should be clear to your reader what you are doing, how you are doing it, and why.

The literature review analyzes the existing research and centres your own work within it. It should provide the reader with a clear understanding of what other people have said about the topic you are investigating. You should make it clear whether the topic you will research is contentious or not, and how much research has been done. Finally, you should explain how this thesis/dissertation will fit within the existing research and what it contributes to the literature overall.

In the methodology section of a social science thesis/dissertation, you should clearly explain how you have performed your research. Did you use qualitative or quantitative methods? How was your process structured? Why did you do it this way? What are the limitations (weaknesses) of your methodological approach?

Once you have explained your methods, it is time to provide your results . What did your research find? This is followed by the discussion , which explores the significance of your results and whether or not they were as you expected. If your research yielded the expected results, why did that happen? If not, why not? Finally, wrap up with a conclusion that reiterates what you did and why it matters, and point to future matters for research. The bibliography section lists all of the sources you cited, and the appendices list any extra information or resources such as raw data, survey questions, etc. that your reader may want to know.

In social science theses/dissertations that rely more heavily on qualitative rather than quantitative methods, the above structure can still be followed. However, sometimes the results and discussion chapters will be intertwined or combined. Certain types of social science theses/dissertations, such as public policy, history, or anthropology, may follow the humanities thesis/dissertation structure as we mentioned above.

Critical steps for writing and structuring a humanities/social science thesis/dissertation

If you are still struggling to get started, here is a checklist of steps for writing and structuring your humanities or social science thesis/dissertation.

  • Choose your thesis/dissertation topic
  • What is the word count/page length requirement?
  • What chapters must be included?
  • What chapters are optional?
  • Conduct preliminary research
  • Decide on your own research methodology
  • Outline your proposed methods and expected results
  • Use your proposed methodology to choose what chapters to include in your thesis/dissertation
  • Create a preliminary table of contents to outline the structure of your thesis/dissertation

By following these steps, you should be able to organize the structure of your humanities or social science thesis/dissertation before you begin writing.

Final tips for writing and structuring a thesis/dissertation

Although writing a thesis/dissertation is a difficult project, it is also very rewarding. You will get the most out of the experience if you properly prepare yourself by carefully learning about each step. Before you decide how to structure your thesis/dissertation, you will need to decide on a thesis topic and come up with a hypothesis. You should do as much preliminary reading and notetaking as you have time for.

Since most people writing a thesis/dissertation are doing it for the first time, you should also take some time to learn about the many tools that exist to help students write better and organize their citations. Citation generators and reference managers like EndNote help you keep track of your sources and AI grammar and writing checkers are helpful as you write. You should also keep in mind that you will need to edit and proofread your thesis/dissertation once you have the bulk of the writing complete. Many thesis editing and proofreading services are available to help you with this as well.

Editor’s pick

Get free updates.

Subscribe to our newsletter for regular insights from the research and publishing industry!

What are the parts of a social science thesis/dissertation? +

A social science thesis/dissertation is usually structured as follows:

How long is a typical social science thesis/dissertation? +

What are the parts of a humanities thesis/dissertation +.

Humanities theses/dissertations are usually structured like this:

  • Thematic Chapters

What is the typical structure of a thematic chapter in a humanities thesis/dissertation? +

A thematic chapter in a humanities thesis/dissertation is structured like this:

How long is a typical humanities thesis/dissertation? +

A typical humanities thesis/dissertation tends to range from 100 to 300 pages in length.

Logo

Tips for writing a PhD dissertation: FAQs answered

From how to choose a topic to writing the abstract and managing work-life balance through the years it takes to complete a doctorate, here we collect expert advice to get you through the PhD writing process

Campus team's avatar

Campus team

Additional links.

  • More on this topic

Figures with light bulb

You may also like

Signposts for help, support, advice and guidance

Popular resources

.css-1txxx8u{overflow:hidden;max-height:81px;text-indent:0px;} Campus webinar: the evolution of interdisciplinarity

A diy guide to starting your own journal, relieve student boredom by ‘activating’ lectures, emotions and learning: what role do emotions play in how and why students learn, can academic prenups help overcome barriers to interdisciplinary research.

Embarking on a PhD is “probably the most challenging task that a young scholar attempts to do”, write Mark Stephan Felix and Ian Smith in their practical guide to dissertation and thesis writing. After years of reading and research to answer a specific question or proposition, the candidate will submit about 80,000 words that explain their methods and results and demonstrate their unique contribution to knowledge. Here are the answers to frequently asked questions about writing a doctoral thesis or dissertation.

What’s the difference between a dissertation and a thesis?

Whatever the genre of the doctorate, a PhD must offer an original contribution to knowledge. The terms “dissertation” and “thesis” both refer to the long-form piece of work produced at the end of a research project and are often used interchangeably. Which one is used might depend on the country, discipline or university. In the UK, “thesis” is generally used for the work done for a PhD, while a “dissertation” is written for a master’s degree. The US did the same until the 1960s, says Oxbridge Essays, when the convention switched, and references appeared to a “master’s thesis” and “doctoral dissertation”. To complicate matters further, undergraduate long essays are also sometimes referred to as a thesis or dissertation.

The Oxford English Dictionary defines “thesis” as “a dissertation, especially by a candidate for a degree” and “dissertation” as “a detailed discourse on a subject, especially one submitted in partial fulfilment of the requirements of a degree or diploma”.

  • Ten platinum rules for PhD supervisors
  • Fostering freedom in PhD students: how supervisors can shape accessible paths for doctoral research
  • Lessons from students on effective research supervision

The title “doctor of philosophy”, incidentally, comes from the degree’s origins, write Dr Felix, an associate professor at Mahidol University in Thailand, and Dr Smith, retired associate professor of education at the University of Sydney , whose co-authored guide focuses on the social sciences. The PhD was first awarded in the 19th century by the philosophy departments of German universities, which at that time taught science, social science and liberal arts.

How long should a PhD thesis be?

A PhD thesis (or dissertation) is typically 60,000 to 120,000 words ( 100 to 300 pages in length ) organised into chapters, divisions and subdivisions (with roughly 10,000 words per chapter) – from introduction (with clear aims and objectives) to conclusion.

The structure of a dissertation will vary depending on discipline (humanities, social sciences and STEM all have their own conventions), location and institution. Examples and guides to structure proliferate online. The University of Salford , for example, lists: title page, declaration, acknowledgements, abstract, table of contents, lists of figures, tables and abbreviations (where needed), chapters, appendices and references.

A scientific-style thesis will likely need: introduction, literature review, materials and methods, results, discussion, bibliography and references.

As well as checking the overall criteria and expectations of your institution for your research, consult your school handbook for the required length and format (font, layout conventions and so on) for your dissertation.

A PhD takes three to four years to complete; this might extend to six to eight years for a part-time doctorate.

What are the steps for completing a PhD?

Before you get started in earnest , you’ll likely have found a potential supervisor, who will guide your PhD journey, and done a research proposal (which outlines what you plan to research and how) as part of your application, as well as a literature review of existing scholarship in the field, which may form part of your final submission.

In the UK, PhD candidates undertake original research and write the results in a thesis or dissertation, says author and vlogger Simon Clark , who posted videos to YouTube throughout his own PhD journey . Then they submit the thesis in hard copy and attend the viva voce (which is Latin for “living voice” and is also called an oral defence or doctoral defence) to convince the examiners that their work is original, understood and all their own. Afterwards, if necessary, they make changes and resubmit. If the changes are approved, the degree is awarded.

The steps are similar in Australia , although candidates are mostly assessed on their thesis only; some universities may include taught courses, and some use a viva voce. A PhD in Australia usually takes three years full time.

In the US, the PhD process begins with taught classes (similar to a taught master’s) and a comprehensive exam (called a “field exam” or “dissertation qualifying exam”) before the candidate embarks on their original research. The whole journey takes four to six years.

A PhD candidate will need three skills and attitudes to get through their doctoral studies, says Tara Brabazon , professor of cultural studies at Flinders University in Australia who has written extensively about the PhD journey :

  • master the academic foundational skills (research, writing, ability to navigate different modalities)
  • time-management skills and the ability to focus on reading and writing
  • determined motivation to do a PhD.

Socrates' methods can still help university student in the battle with misinformation

How do I choose the topic for my PhD dissertation or thesis?

It’s important to find a topic that will sustain your interest for the years it will take to complete a PhD. “Finding a sustainable topic is the most important thing you [as a PhD student] would do,” says Dr Brabazon in a video for Times Higher Education . “Write down on a big piece of paper all the topics, all the ideas, all the questions that really interest you, and start to cross out all the ones that might just be a passing interest.” Also, she says, impose the “Who cares? Who gives a damn?” question to decide if the topic will be useful in a future academic career.

The availability of funding and scholarships is also often an important factor in this decision, says veteran PhD supervisor Richard Godwin, from Harper Adams University .

Define a gap in knowledge – and one that can be questioned, explored, researched and written about in the time available to you, says Gina Wisker, head of the Centre for Learning and Teaching at the University of Brighton. “Set some boundaries,” she advises. “Don’t try to ask everything related to your topic in every way.”

James Hartley, research professor in psychology at Keele University, says it can also be useful to think about topics that spark general interest. If you do pick something that taps into the zeitgeist, your findings are more likely to be noticed.

You also need to find someone else who is interested in it, too. For STEM candidates , this will probably be a case of joining a team of people working in a similar area where, ideally, scholarship funding is available. A centre for doctoral training (CDT) or doctoral training partnership (DTP) will advertise research projects. For those in the liberal arts and social sciences, it will be a matter of identifying a suitable supervisor .

Avoid topics that are too broad (hunger across a whole country, for example) or too narrow (hunger in a single street) to yield useful solutions of academic significance, write Mark Stephan Felix and Ian Smith. And ensure that you’re not repeating previous research or trying to solve a problem that has already been answered. A PhD thesis must be original.

What is a thesis proposal?

After you have read widely to refine your topic and ensure that it and your research methods are original, and discussed your project with a (potential) supervisor, you’re ready to write a thesis proposal , a document of 1,500 to 3,000 words that sets out the proposed direction of your research. In the UK, a research proposal is usually part of the application process for admission to a research degree. As with the final dissertation itself, format varies among disciplines, institutions and countries but will usually contain title page, aims, literature review, methodology, timetable and bibliography. Examples of research proposals are available online.

How to write an abstract for a dissertation or thesis

The abstract presents your thesis to the wider world – and as such may be its most important element , says the NUI Galway writing guide. It outlines the why, how, what and so what of the thesis . Unlike the introduction, which provides background but not research findings, the abstract summarises all sections of the dissertation in a concise, thorough, focused way and demonstrates how well the writer understands their material. Check word-length limits with your university – and stick to them. About 300 to 500 words is a rough guide ­– but it can be up to 1,000 words.

The abstract is also important for selection and indexing of your thesis, according to the University of Melbourne guide , so be sure to include searchable keywords.

It is the first thing to be read but the last element you should write. However, Pat Thomson , professor of education at the University of Nottingham , advises that it is not something to be tackled at the last minute.

How to write a stellar conclusion

As well as chapter conclusions, a thesis often has an overall conclusion to draw together the key points covered and to reflect on the unique contribution to knowledge. It can comment on future implications of the research and open up new ideas emanating from the work. It is shorter and more general than the discussion chapter , says online editing site Scribbr, and reiterates how the work answers the main question posed at the beginning of the thesis. The conclusion chapter also often discusses the limitations of the research (time, scope, word limit, access) in a constructive manner.

It can be useful to keep a collection of ideas as you go – in the online forum DoctoralWriting SIG , academic developer Claire Aitchison, of the University of South Australia , suggests using a “conclusions bank” for themes and inspirations, and using free-writing to keep this final section fresh. (Just when you feel you’ve run out of steam.) Avoid aggrandising or exaggerating the impact of your work. It should remind the reader what has been done, and why it matters.

How to format a bibliography (or where to find a reliable model)

Most universities use a preferred style of references , writes THE associate editor Ingrid Curl. Make sure you know what this is and follow it. “One of the most common errors in academic writing is to cite papers in the text that do not then appear in the bibliography. All references in your thesis need to be cross-checked with the bibliography before submission. Using a database during your research can save a great deal of time in the writing-up process.”

A bibliography contains not only works cited explicitly but also those that have informed or contributed to the research – and as such illustrates its scope; works are not limited to written publications but include sources such as film or visual art.

Examiners can start marking from the back of the script, writes Dr Brabazon. “Just as cooks are judged by their ingredients and implements, we judge doctoral students by the calibre of their sources,” she advises. She also says that candidates should be prepared to speak in an oral examination of the PhD about any texts included in their bibliography, especially if there is a disconnect between the thesis and the texts listed.

Can I use informal language in my PhD?

Don’t write like a stereotypical academic , say Kevin Haggerty, professor of sociology at the University of Alberta , and Aaron Doyle, associate professor in sociology at Carleton University , in their tongue-in-cheek guide to the PhD journey. “If you cannot write clearly and persuasively, everything about PhD study becomes harder.” Avoid jargon, exotic words, passive voice and long, convoluted sentences – and work on it consistently. “Writing is like playing guitar; it can improve only through consistent, concerted effort.”

Be deliberate and take care with your writing . “Write your first draft, leave it and then come back to it with a critical eye. Look objectively at the writing and read it closely for style and sense,” advises THE ’s Ms Curl. “Look out for common errors such as dangling modifiers, subject-verb disagreement and inconsistency. If you are too involved with the text to be able to take a step back and do this, then ask a friend or colleague to read it with a critical eye. Remember Hemingway’s advice: ‘Prose is architecture, not interior decoration.’ Clarity is key.”

How often should a PhD candidate meet with their supervisor?

Since the PhD supervisor provides a range of support and advice – including on research techniques, planning and submission – regular formal supervisions are essential, as is establishing a line of contact such as email if the candidate needs help or advice outside arranged times. The frequency varies according to university, discipline and individual scholars.

Once a week is ideal, says Dr Brabazon. She also advocates a two-hour initial meeting to establish the foundations of the candidate-supervisor relationship .

The University of Edinburgh guide to writing a thesis suggests that creating a timetable of supervisor meetings right at the beginning of the research process will allow candidates to ensure that their work stays on track throughout. The meetings are also the place to get regular feedback on draft chapters.

“A clear structure and a solid framework are vital for research,” writes Dr Godwin on THE Campus . Use your supervisor to establish this and provide a realistic view of what can be achieved. “It is vital to help students identify the true scientific merit, the practical significance of their work and its value to society.”

How to proofread your dissertation (what to look for)

Proofreading is the final step before printing and submission. Give yourself time to ensure that your work is the best it can be . Don’t leave proofreading to the last minute; ideally, break it up into a few close-reading sessions. Find a quiet place without distractions. A checklist can help ensure that all aspects are covered.

Proofing is often helped by a change of format – so it can be easier to read a printout rather than working off the screen – or by reading sections out of order. Fresh eyes are better at spotting typographical errors and inconsistencies, so leave time between writing and proofreading. Check with your university’s policies before asking another person to proofread your thesis for you.

As well as close details such as spelling and grammar, check that all sections are complete, all required elements are included , and nothing is repeated or redundant. Don’t forget to check headings and subheadings. Does the text flow from one section to another? Is the structure clear? Is the work a coherent whole with a clear line throughout?

Ensure consistency in, for example, UK v US spellings, capitalisation, format, numbers (digits or words, commas, units of measurement), contractions, italics and hyphenation. Spellchecks and online plagiarism checkers are also your friend.

Image of students being support to cross a gap in their learning

How do you manage your time to complete a PhD dissertation?

Treat your PhD like a full-time job, that is, with an eight-hour working day. Within that, you’ll need to plan your time in a way that gives a sense of progress . Setbacks and periods where it feels as if you are treading water are all but inevitable, so keeping track of small wins is important, writes A Happy PhD blogger Luis P. Prieto.

Be specific with your goals – use the SMART acronym (specific, measurable, attainable, relevant and timely).

And it’s never too soon to start writing – even if early drafts are overwritten and discarded.

“ Write little and write often . Many of us make the mistake of taking to writing as one would take to a sprint, in other words, with relatively short bursts of intense activity. Whilst this can prove productive, generally speaking it is not sustainable…In addition to sustaining your activity, writing little bits on a frequent basis ensures that you progress with your thinking. The comfort of remaining in abstract thought is common; writing forces us to concretise our thinking,” says Christian Gilliam, AHSS researcher developer at the University of Cambridge ’s Centre for Teaching and Learning.

Make time to write. “If you are more alert early in the day, find times that suit you in the morning; if you are a ‘night person’, block out some writing sessions in the evenings,” advises NUI Galway’s Dermot Burns, a lecturer in English and creative arts. Set targets, keep daily notes of experiment details that you will need in your thesis, don’t confuse writing with editing or revising – and always back up your work.

What work-life balance tips should I follow to complete my dissertation?

During your PhD programme, you may have opportunities to take part in professional development activities, such as teaching, attending academic conferences and publishing your work. Your research may include residencies, field trips or archive visits. This will require time-management skills as well as prioritising where you devote your energy and factoring in rest and relaxation. Organise your routine to suit your needs , and plan for steady and regular progress.

How to deal with setbacks while writing a thesis or dissertation

Have a contingency plan for delays or roadblocks such as unexpected results.

Accept that writing is messy, first drafts are imperfect, and writer’s block is inevitable, says Dr Burns. His tips for breaking it include relaxation to free your mind from clutter, writing a plan and drawing a mind map of key points for clarity. He also advises feedback, reflection and revision: “Progressing from a rough version of your thoughts to a superior and workable text takes time, effort, different perspectives and some expertise.”

“Academia can be a relentlessly brutal merry-go-round of rejection, rebuttal and failure,” writes Lorraine Hope , professor of applied cognitive psychology at the University of Portsmouth, on THE Campus. Resilience is important. Ensure that you and your supervisor have a relationship that supports open, frank, judgement-free communication.

If you would like advice and insight from academics and university staff delivered direct to your inbox each week, sign up for the Campus newsletter .

Authoring a PhD Thesis: How to Plan, Draft, Write and Finish a Doctoral Dissertation (2003), by Patrick Dunleavy

Writing Your Dissertation in Fifteen Minutes a Day: A Guide to Starting, Revising, and Finishing Your Doctoral Thesis (1998), by Joan Balker

Challenges in Writing Your Dissertation: Coping with the Emotional, Interpersonal, and Spiritual Struggles (2015), by Noelle Sterne

Campus webinar: the evolution of interdisciplinarity

Using data skills to turn students’ passion for sports into rewarding careers, contextual learning: linking learning to the real world, how mozart and the arts can help students feel less awkward, strategies to empower first-generation university students, how self-determination theory supports post-covid students.

Register for free

and unlock a host of features on the THE site

Get the Reddit app

A subreddit dedicated to PhDs.

How long did you take your write your dissertation?

Need some confirmation bias here!

By continuing, you agree to our User Agreement and acknowledge that you understand the Privacy Policy .

Enter the 6-digit code from your authenticator app

You’ve set up two-factor authentication for this account.

Enter a 6-digit backup code

Create your username and password.

Reddit is anonymous, so your username is what you’ll go by here. Choose wisely—because once you get a name, you can’t change it.

Reset your password

Enter your email address or username and we’ll send you a link to reset your password

Check your inbox

An email with a link to reset your password was sent to the email address associated with your account

Choose a Reddit account to continue

  • Formatting Your Dissertation
  • Introduction

Harvard Griffin GSAS strives to provide students with timely, accurate, and clear information. If you need help understanding a specific policy, please contact the office that administers that policy.

  • Application for Degree
  • Credit for Completed Graduate Work
  • Ad Hoc Degree Programs
  • Acknowledging the Work of Others
  • Dissertation Advisory Committee
  • Publishing Options
  • Subject, Invention, and Patents
  • Submitting Your Dissertation
  • English Language Proficiency
  • PhD Program Requirements
  • Secondary Fields
  • Year of Graduate Study (G-Year)
  • Master's Degrees
  • Grade and Examination Requirements
  • Conduct and Safety
  • Financial Aid
  • Non-Resident Students
  • Registration
  • Residence Halls
  • Student Groups

When preparing the dissertation for submission, students must meet the following minimum formatting requirements. The Registrar’s Office will review the dissertation for compliance and these formatting elements and will contact the student to confirm acceptance or to request revision. The Harvard Griffin GSAS resource on dissertation formatting best practices expands on many of the elements below.

Please carefully review your dissertation before submitting it to ProQuestETD. The Registrar’s Office will email you through ProQuest if they have identified major formatting errors that need correction. Students will be provided with a brief extended deadline to make only the requested formatting updates.  

  • Embedded Fonts : If fonts are not embedded, non-English characters may not appear as intended. It is the student’s responsibility to make sure that fonts are embedded properly prior to submission. Instructions for embedding fonts can be found on the Dissertation Formatting Guidance resource .  
  • Thesis Acceptance Certificate: A copy of the Thesis Acceptance Certificate (TAC) should appear as the first page. This page should not be counted or numbered. The TAC will appear in the online version of the published dissertation. The author name and date on the TAC and title page should be the same.  
  • Title Page: The dissertation begins with the title page; the title should be as concise as possible and should provide an accurate description of the dissertation. The author name and date on the TAC and title page should be the same. Do not print a page number on the title page. It is understood to be page  i  for counting purposes only. 
  • Abstract : An abstract, numbered as page  iii , should immediately follow the copyright page and should state the problem, describe the methods and procedures used, and give the main results or conclusions of the research. The abstract will appear in the online version of the dissertation and will be made available by ProQuest and DASH. There is no maximum word count for the abstract.  
  • Preliminary pages (abstract, table of contents, list of tables, graphs, illustrations, and preface) should use small Roman numerals (i, ii, iii, iv, v, etc.). 
  • All pages must contain text or images.  
  • Count the title page as page i and the copyright page as page ii, but do not print page numbers on either page. 
  • For the body of text, use Arabic numbers (1, 2, 3, 4, 5, etc.) starting with page 1 on the first page of text.
  • Page numbers must be centered throughout the manuscript at the top or bottom. 
  • Every numbered page must be consecutively ordered, including tables, graphs, illustrations, and bibliography/index (if included); letter suffixes (such as 10a, 10b, etc.) are not allowed. 
  • It is customary not to have a page number on the page containing a chapter heading. Check pagination carefully. Account for all pages. 
  • Copyright Statement: A copyright notice should appear on a separate page immediately following the title page and include the copyright symbol ©, the year of first publication of the work, and the name of the author: © [ year ] [ Author’s Name ]. All rights reserved. Alternatively, students may choose to license their work openly under a Creative Commons license. The author remains the copyright holder while at the same time granting upfront permission to others to read, share, and—depending on the license—adapt the work so long as proper attribution is given. (If a student chooses a Creative Commons license, the copyright statement must not include the “all rights reserved” disclaimer and should instead indicate the specific Creative Commons license.) Please note: The copyright statement applies only to the student’s own work; the copyright status of third-party material incorporated into the dissertation will not change. Do not  print a page number on the copyright page. It is understood to be page  ii  for counting purposes only. 
  • Abstract 
  • Table of Contents 
  • Front Matter 
  • Body of Text 
  • Back Matter 

Students can refer to the resource on Dissertation Formatting Best Practice Resource for information on best practices for front and back matter

Individual academic programs may require additional formatting elements to meet the standards of a specific field or discipline. Students are responsible to ensure that their Dissertation Advisory Committee is in support of the final formatting as signified by the sign off on the Thesis Acceptance Certificate. Any deviation from these requirements may lead to rejection of the dissertation and delay in the conferral of the degree. 

CONTACT INFO

Katie riggs, academic programs, explore events.

  • Alumni and Friends
  • Directories
  • News and Events
  • Prospective Student Information

Search form

  • International Students
  • Non-Degree Visiting Students
  • Newly Admitted Students
  • Financial Aid
  • Student Registration Policies
  • Student Employment
  • Satisfactory Academic Progress
  • Incomplete Coursework
  • Language Exam Requirements
  • Joint Degree Programs
  • Transfer Course Credits

Dissertation and Defense

  • Information on Academic Disputes and Formal Grievance Policy
  • Leave of Absence and Withdrawal
  • Faculty Departures: Impact on Student Advising
  • Academic Requirements and Milestones
  • Course Enrollment
  • Receiving Your Degree
  • Resources for Students
  • BA/MA programs in the Humanities
  • Student Leadership Opportunities

Dissertation Committee

Dissertation defense process.

The dissertation defense is one of the most important moments in a graduate student’s career. When a student is deemed to be ready to defend the doctoral thesis, the student is the expert in a particular research area.  At the defense the student is expected to cogently and clearly explain their research and how it fits with other scholarship in the field of study. The exact nature of the oral defense may vary by discipline and department, so it is important that the student talk to the dissertation committee chair about what to expect and how to prepare.

The defense typically lasts about two hours and may be open to the public. The defense aims to accomplish two goals. First, it will provide an occasion for the presentation and recognition of the completed doctoral work. Second, it will furnish the opportunity for discussion and formal evaluation of the dissertation.

The timing of the defense will be set by the student in consultation with the dissertation committee. Students should contact their departments to schedule the dissertation defense. Note that some departments do not schedule examinations in summer. The student should send copies of the dissertation to the members of the committee no later than three weeks prior to the defense. The time and place of the defense shall be announced on the department’s calendar of events.  

The chair of the student's dissertation committee and at least one other committee member must be present in person. In the case that a committee member is unable to attend in person, the committee member may participate by Skype or other teleconference media. The defense must be open to all faculty within the department. Broader attendance will be left to the discretion of the student in consultation with the committee.

Please note: Departments will be allowed to have in-person defenses. Additionally, there is no reason to limit remote defenses if that is what the participants want and the department/committee approves. 

The dissertation defense shall be an open presentation by the student on the main aspects of the research reported in the dissertation, followed by questions, comments, and discussion. The chair of the dissertation committee or a designee shall act as the moderator of this discussion and shall have discretion to decide whether questions are germane to the topic of the dissertation. After discussion is completed, the departmental faculty in attendance and the dissertation committee will decide privately whether the dissertation defense has been satisfactory.

The dissertation chair or designee will report the results as soon as possible after the dissertation defense to the Dean of Students office by submitting a signed Report of Final Examination and a Dissertation Defense Form. If the manuscript is to undergo revisions, the student will be informed of the timeframe in which the revisions are to be completed.

Dissertation Submission

A final copy of the dissertation must be submitted following the procedure outlined on the  Dissertation Office  website by Friday of the seventh week of the quarter in which a student is graduating, sixth week in summer quarter. The defense should be held in sufficient advance of this deadline to allow revisions recommended by the student's dissertation committee.

Students should contact the Dissertation Office no later than early in the quarter in which they intend to graduate in order to resolve any questions or problems they may have in preparing the dissertation. The Dissertation Office staff is willing to look at a draft of the dissertation. Having them do so avoids both the expense and headache of having to re-format the dissertation. They offer a schedule to help students plan for a smooth process to meet these deadlines.

Additional requirements including where the dissertation must be submitted is available at: https://studentmanual.uchicago.edu/academic-policies/dissertation-requirements/

Office of the Dean of Students [email protected] 773.702.1552

how long is a humanities dissertation

  • How Long Is a PhD Thesis?
  • Doing a PhD

It’s no secret that one of the most challenging aspects of a PhD degree is the volume of work that goes into writing your thesis . So this raises the question, exactly how long is a thesis?

Unfortunately, there’s no one size fits all answer to this question. However, from the analysis of over 100 PhD theses, the average thesis length is between 80,000 and 100,000 words. A further analysis of 1000 PhD thesis shows the average number of pages to be 204 . In reality, the actual word count for each PhD thesis will depend on the specific subject and the university it is being hosted by. This is because universities set their own word length requirements, with most found to be opting for around 100,000.

To find out more about how these word limits differ between universities, how the average word count from STEM thesis differ from non-STEM thesis and a more detailed breakdown from the analysis of over 1000 PhDs, carry on reading the below.

Word Count Differences Between Universities

For any PhD student writing a thesis, they will find that their document will be subject to a word limit set by their university. In nearly all cases, the limit only concerns the maximum number of words and doesn’t place any restrictions on the minimum word limit. The reason for this is that the student will be expected to write their thesis with the aim of clearly explaining their research, and so it is up to the student to determine what he deems appropriate.

Saying this, it is well accepted amongst PhD students and supervisors that the absence of a lower limit doesn’t suggest that a thesis can be ‘light’. Your thesis will focus on several years worth of original research and explore new ideas, theories or concepts. Besides this, your thesis will need to cover a wide range of topics such as your literature review, research methodology, results and conclusion. Therefore, your examiners will expect the length of your thesis to be proportional to convey all this information to a sufficient level.

Selecting a handful of universities at random, they state the following thesis word limits on their website:

  • University of Edinburgh: 100,000
  • University of Exeter: 100,000
  • University of Leister: 80,000
  • University of Bath: 80,000
  • University of Warwick: 70,000

The above universities set upper word limits that apply across the board, however, some universities, such as the University of Birmingham and the University of Sheffield, set different word limits for different departments. For example, the University of Sheffield adopts these limits:

  • Arts & Humanities: 75,000
  • Medicine, Dentistry & Health: 75,000
  • Science: 80,000
  • Social Sciences: 75,000-100,000

Although there’s a range of limit, it’s safe to say that the majority fall within the 80,000 to 100,000 bracket.

Word Count Based on Data from past Theses

A poll of 149 postdocs.

In mid-2019, Dr Eva Lantsoght, a published author, academic blogger and Structural Engineering Professor, conducted a poll which asked postgraduate doctoral students to share the length of their final thesis. 149 PostDoc students responded to the survey, with the majority reporting a length falling within the ‘80,000 – 120,000 words’ bracket as seen below.

DiscoverPhDs_How-long-is-a-PhD-Thesis_Poll

Analysis of 1000 PhD Theses

Over a three-year time period, Dr Ian Brailsford, a then Postgraduate Learning Adviser at the University of Auckland, analysed 1000 doctoral thesis submitted to his university’s library. The PhD theses which formed the basis of his analysis were produced between 2008 to 2017 and showed:

  • Average number of pages = 204
  • Median number of pages = 198
  • Average number of chapters = 7.6

We should note that the above metrics only cover the content falling within the main body of the thesis. This includes the introduction, literature review, methods section, results chapter, discussions and conclusions. All other sections, such as the title page, abstract, table of contents, acknowledgements, bibliography and appendices were omitted from the count.

Although it’s impossible to draw the exact word count from the number of pages alone, by using the universities recommended format of 12pt Times New Roman and 1.5 lines spacing, and assuming 10% of the main body are figures and footnotes, this equates to an average main body of 52,000 words.

STEM vs Non-STEM

As part of Dr Ian Brailsford’s analysis, he also compared the length of STEM doctorate theses to non-STEM theses. He found that STEM theses tended to be shorter. In fact, he found STEM theses to have a medium page length of 159 whilst non-STEM theses had a medium of around 223 pages. This is a 40% increase in average length!

Can You Exceed the Word Count?

Whilst most universities will allow you to go over the word count if you need to, it comes with the caveat that you must have a very strong reason for needing to do so. Besides this, your supervisor will also need to support your request. This is to acknowledge that they have reviewed your situation and agree that exceeding the word limit will be absolutely necessary to avoid detriment unnecessary detriment to your work.

This means that whilst it is possible to submit a thesis over 100,000 words or more, it’s unlikely that your research project will need to.

How Does This Compare to a Masters Dissertation?

The average Masters dissertation length is approximately 20,000 words whilst a thesis is 4 to 5 times this length at approximately 80,000 – 100,000.

The key reason for this difference is because of the level of knowledge they convey. A Master’s dissertation focuses on concluding from existing knowledge whilst a PhD thesis focuses on drawing a conclusion from new knowledge. As a result, the thesis is significantly longer as the new knowledge needs to be well documented so it can be verified, disseminated and used to shape future research.

Finding a PhD has never been this easy – search for a PhD by keyword, location or academic area of interest.

Related Reading

Unfortunately, the completion of your thesis doesn’t mark the end of your degree just yet. Once you submit your thesis, it’s time to start preparing for your viva – the all-to-fun thesis defence interview! To help you prepare for this, we’ve produced a helpful guide which you can read here: The Complete Guide to PhD Vivas.

Browse PhDs Now

Join thousands of students.

Join thousands of other students and stay up to date with the latest PhD programmes, funding opportunities and advice.

Have a language expert improve your writing

Run a free plagiarism check in 10 minutes, automatically generate references for free.

  • Knowledge Base
  • Dissertation

What Is a Dissertation? | 5 Essential Questions to Get Started

Published on 26 March 2020 by Jack Caulfield . Revised on 5 May 2022.

A dissertation is a large research project undertaken at the end of a degree. It involves in-depth consideration of a problem or question chosen by the student. It is usually the largest (and final) piece of written work produced during a degree.

The length and structure of a dissertation vary widely depending on the level and field of study. However, there are some key questions that can help you understand the requirements and get started on your dissertation project.

Instantly correct all language mistakes in your text

Be assured that you'll submit flawless writing. Upload your document to correct all your mistakes.

upload-your-document-ai-proofreader

Table of contents

When and why do you have to write a dissertation, who will supervise your dissertation, what type of research will you do, how should your dissertation be structured, what formatting and referencing rules do you have to follow, frequently asked questions about dissertations.

A dissertation, sometimes called a thesis, comes at the end of an undergraduate or postgraduate degree. It is a larger project than the other essays you’ve written, requiring a higher word count and a greater depth of research.

You’ll generally work on your dissertation during the final year of your degree, over a longer period than you would take for a standard essay . For example, the dissertation might be your main focus for the last six months of your degree.

Why is the dissertation important?

The dissertation is a test of your capacity for independent research. You are given a lot of autonomy in writing your dissertation: you come up with your own ideas, conduct your own research, and write and structure the text by yourself.

This means that it is an important preparation for your future, whether you continue in academia or not: it teaches you to manage your own time, generate original ideas, and work independently.

Prevent plagiarism, run a free check.

During the planning and writing of your dissertation, you’ll work with a supervisor from your department. The supervisor’s job is to give you feedback and advice throughout the process.

The dissertation supervisor is often assigned by the department, but you might be allowed to indicate preferences or approach potential supervisors. If so, try to pick someone who is familiar with your chosen topic, whom you get along with on a personal level, and whose feedback you’ve found useful in the past.

How will your supervisor help you?

Your supervisor is there to guide you through the dissertation project, but you’re still working independently. They can give feedback on your ideas, but not come up with ideas for you.

You may need to take the initiative to request an initial meeting with your supervisor. Then you can plan out your future meetings and set reasonable deadlines for things like completion of data collection, a structure outline, a first chapter, a first draft, and so on.

Make sure to prepare in advance for your meetings. Formulate your ideas as fully as you can, and determine where exactly you’re having difficulties so you can ask your supervisor for specific advice.

Your approach to your dissertation will vary depending on your field of study. The first thing to consider is whether you will do empirical research , which involves collecting original data, or non-empirical research , which involves analysing sources.

Empirical dissertations (sciences)

An empirical dissertation focuses on collecting and analysing original data. You’ll usually write this type of dissertation if you are studying a subject in the sciences or social sciences.

  • What are airline workers’ attitudes towards the challenges posed for their industry by climate change?
  • How effective is cognitive behavioural therapy in treating depression in young adults?
  • What are the short-term health effects of switching from smoking cigarettes to e-cigarettes?

There are many different empirical research methods you can use to answer these questions – for example, experiments , observations, surveys , and interviews.

When doing empirical research, you need to consider things like the variables you will investigate, the reliability and validity of your measurements, and your sampling method . The aim is to produce robust, reproducible scientific knowledge.

Non-empirical dissertations (arts and humanities)

A non-empirical dissertation works with existing research or other texts, presenting original analysis, critique and argumentation, but no original data. This approach is typical of arts and humanities subjects.

  • What attitudes did commentators in the British press take towards the French Revolution in 1789–1792?
  • How do the themes of gender and inheritance intersect in Shakespeare’s Macbeth ?
  • How did Plato’s Republic and Thomas More’s Utopia influence nineteenth century utopian socialist thought?

The first steps in this type of dissertation are to decide on your topic and begin collecting your primary and secondary sources .

Primary sources are the direct objects of your research. They give you first-hand evidence about your subject. Examples of primary sources include novels, artworks and historical documents.

Secondary sources provide information that informs your analysis. They describe, interpret, or evaluate information from primary sources. For example, you might consider previous analyses of the novel or author you are working on, or theoretical texts that you plan to apply to your primary sources.

Dissertations are divided into chapters and sections. Empirical dissertations usually follow a standard structure, while non-empirical dissertations are more flexible.

Structure of an empirical dissertation

Empirical dissertations generally include these chapters:

  • Introduction : An explanation of your topic and the research question(s) you want to answer.
  • Literature review : A survey and evaluation of previous research on your topic.
  • Methodology : An explanation of how you collected and analysed your data.
  • Results : A brief description of what you found.
  • Discussion : Interpretation of what these results reveal.
  • Conclusion : Answers to your research question(s) and summary of what your findings contribute to knowledge in your field.

Sometimes the order or naming of chapters might be slightly different, but all of the above information must be included in order to produce thorough, valid scientific research.

Other dissertation structures

If your dissertation doesn’t involve data collection, your structure is more flexible. You can think of it like an extended essay – the text should be logically organised in a way that serves your argument:

  • Introduction: An explanation of your topic and the question(s) you want to answer.
  • Main body: The development of your analysis, usually divided into 2–4 chapters.
  • Conclusion: Answers to your research question(s) and summary of what your analysis contributes to knowledge in your field.

The chapters of the main body can be organised around different themes, time periods, or texts. Below you can see some example structures for dissertations in different subjects.

  • Political philosophy

This example, on the topic of the British press’s coverage of the French Revolution, shows how you might structure each chapter around a specific theme.

Example of a dissertation structure in history

This example, on the topic of Plato’s and More’s influences on utopian socialist thought, shows a different approach to dividing the chapters by theme.

Example of a dissertation structure in political philosophy

This example, a master’s dissertation on the topic of how writers respond to persecution, shows how you can also use section headings within each chapter. Each of the three chapters deals with a specific text, while the sections are organised thematically.

Example of a dissertation structure in literature

Like other academic texts, it’s important that your dissertation follows the formatting guidelines set out by your university. You can lose marks unnecessarily over mistakes, so it’s worth taking the time to get all these elements right.

Formatting guidelines concern things like:

  • line spacing
  • page numbers
  • punctuation
  • title pages
  • presentation of tables and figures

If you’re unsure about the formatting requirements, check with your supervisor or department. You can lose marks unnecessarily over mistakes, so it’s worth taking the time to get all these elements right.

How will you reference your sources?

Referencing means properly listing the sources you cite and refer to in your dissertation, so that the reader can find them. This avoids plagiarism by acknowledging where you’ve used the work of others.

Keep track of everything you read as you prepare your dissertation. The key information to note down for a reference is:

  • The publication date
  • Page numbers for the parts you refer to (especially when using direct quotes)

Different referencing styles each have their own specific rules for how to reference. The most commonly used styles in UK universities are listed below.

&
An author–date citation in brackets in the text… …corresponding to an entry in the alphabetised reference list at the end.
A superscript or bracketed reference number in the text… …corresponding to an entry in the numbered reference list at the end.
A footnote in the text that gives full source information… …and an alphabetised bibliography at the end listing all sources.

You can use the free APA Reference Generator to automatically create and store your references.

APA Reference Generator

The words ‘ dissertation ’ and ‘thesis’ both refer to a large written research project undertaken to complete a degree, but they are used differently depending on the country:

  • In the UK, you write a dissertation at the end of a bachelor’s or master’s degree, and you write a thesis to complete a PhD.
  • In the US, it’s the other way around: you may write a thesis at the end of a bachelor’s or master’s degree, and you write a dissertation to complete a PhD.

The main difference is in terms of scale – a dissertation is usually much longer than the other essays you complete during your degree.

Another key difference is that you are given much more independence when working on a dissertation. You choose your own dissertation topic , and you have to conduct the research and write the dissertation yourself (with some assistance from your supervisor).

Dissertation word counts vary widely across different fields, institutions, and levels of education:

  • An undergraduate dissertation is typically 8,000–15,000 words
  • A master’s dissertation is typically 12,000–50,000 words
  • A PhD thesis is typically book-length: 70,000–100,000 words

However, none of these are strict guidelines – your word count may be lower or higher than the numbers stated here. Always check the guidelines provided by your university to determine how long your own dissertation should be.

At the bachelor’s and master’s levels, the dissertation is usually the main focus of your final year. You might work on it (alongside other classes) for the entirety of the final year, or for the last six months. This includes formulating an idea, doing the research, and writing up.

A PhD thesis takes a longer time, as the thesis is the main focus of the degree. A PhD thesis might be being formulated and worked on for the whole four years of the degree program. The writing process alone can take around 18 months.

Cite this Scribbr article

If you want to cite this source, you can copy and paste the citation or click the ‘Cite this Scribbr article’ button to automatically add the citation to our free Reference Generator.

Caulfield, J. (2022, May 05). What Is a Dissertation? | 5 Essential Questions to Get Started. Scribbr. Retrieved 19 August 2024, from https://www.scribbr.co.uk/thesis-dissertation/what-is-a-dissertation/

Is this article helpful?

Jack Caulfield

Jack Caulfield

Other students also liked, how to choose a dissertation topic | 8 steps to follow, how to write a dissertation proposal | a step-by-step guide, what is a literature review | guide, template, & examples.

Banner Image

Library Guides

Dissertations 2: structure: thematic.

In the humanities, a thematic dissertation is often structured like a long essay. It can contain: 

Title page 

Abstract 

Table of contents 

Introduction  

Literature review (which can be included in the introduction rather than as a separate chapter. Check with your supervisor if you are unsure). 

Theme 1 

Theme 2 

Theme 3 

Conclusion 

Bibliography 

Appendices 

Abstracts are used by other researchers to establish the relevance of the study to their own work. Therefore, they should contain the what, why, who, where and how of your project.  

They are typically between 250 – 300 words long, offer a summary of the main findings and present the conclusions, so you should attempt to write an abstract (if requested), after you have finished writing the dissertation.  

A typical abstract summarises: 

What the study aimed to achieve  

The methodology used 

Why the research was conducted 

Why the research is important 

Who/what was researched 

Table of Contents

The table of contents should list all the items included in your dissertation.  

It is a good idea to use the electronic table of contents feature in Word to automatically link it to your chapter headings and page numbers. Attempting to manually create a table of contents means that you will have to adjust your page numbers every time you edit your work before submission, which may waste valuable time!  

This useful  video will walk you through the formatting of longer documents using the electronic table of contents feature. 

Introduction

The introduction explains the how, what, where, when, why and who of the research. It introduces the reader to your dissertation and should act as a clear guide as to what it will cover.  

The introduction may include the following content: 

Introduce the topic of the dissertation

  • State why the topic is of interest 
  • Give background information on the subject. 
  • Refer to the main debates in the field

Identify the scope of your research 

  • Highlight what hasn't already been said by the literature  
  • Demonstrate what you seek to investigate, and why 
  • Present the aim of the dissertation. 
  • Mention your research question or hypothesis 

Indicate your approach  

  • Introduce your main argument (especially if you have a research question, rather than hypothesis). 
  • Mention your methods/research design. 
  • Outline the dissertation structure (introduce the main points that you will discuss in the order they will be presented). 

Normally, the introduction is roughly 10% of a dissertation word count. 

Literature Review

The term “literature” in “literature review” comprises scholarly articles, books, and other sources (e.g. reports) relevant to a particular issue, area of research or theory. In a dissertation, the literature review illustrates what the literature already says on your research subject, providing summary and synthesis of such literature.  

It is generally structured by topic, starting from general background and concepts, and then addressing what can be found - and cannot be found - on the specific focus of your dissertation. Indeed, the literature review should identify gaps in the literature, that your research aims to fill. This requires you to engage critically with the literature, not merely reproduce the critical understanding of others.  

In sum, literature reviews should demonstrate how your research question can be located in a wider field of inquiry. Therefore, a literature review needs to address the connections between your work and the work of others by highlighting links between them. In doing so, you will demonstrate the foundations of your project and show how you are taking the line of inquiry forwards.  

By the end of your literature review, your reader should be able to see: 

The gap in knowledge and understanding which you say exists in the field. 

How your research question will work within that gap. 

The work other researchers have carried out and the issues debated in the field. 

That you have a good understanding of the field and that you are critically engaged with the debates (Burnett, 2009). 

For more detailed guidance on how to write literature reviews, check out the Literature Review Guide. 

Theme Chapters

In a thematic structure, the core chapters present analysis and discussion of different themes relevant to answer the research question and support the overall argument of the dissertation. The chapters will include analysis of texts/ research material. They can explore and connect academic theories/research to develop an argument. Stella Cottrell offers some good guidance on how to structure your theme chapters. Each chapter should have the following elements (Cottrell, 2014, p183):

Theme: What is the theme of this chapter? Sequence your themes logically (e.g. from general to specific).  

Argument: What argument does this chapter present?  

Material: What material you will be using for this chapter? 

Clustering: What are the main points you want to make? Deal with one point at a time, and don't jum around? Dedicate your points to sub-headings and paragraphs.  

Sequence: In what order are you going to present the points you want to make in this chapter? Draw an outline of the chapter before starting writing it.  

Introduction and Conclusion: Each chapter should have a short introduction and conclusion. 

The conclusion is the final chapter of your dissertation. It should flow logically from the previously presented text; therefore, you should avoid introducing new ideas, new data, or a new direction.  

Ideally, the conclusion should leave the reader with a clear understanding of the discovery or argument you have advanced.  

This can be done by: 

Summarising and synthesising your main findings and how they relate to your research question or hypotheses  

Demonstrating the relevance and importance of your work in the wider context of your field. For example, what recommendations would you make for future research? What do we know now that we didn’t know before? 

Link your conclusion to your introduction as both frame your dissertation. 

A conclusion is roughly five to ten percent of the word count of the dissertation. 

Avoid excessive detail. Decide what your reader needs to know. 

Don’t introduce any new information such as theories, data or ideas.  

Sum up the main points of your research.  

Bibliography

While writing your dissertation, you would have referred to the works and research of many different authors and editors in your field of study. These works should be acknowledged in the bibliography where you will list writers alphabetically by surname. 

For example: 

Poloian, L.R. (2013).  Retailing principles: global, multichannel, and managerial viewpoints.  New York: Fairchild.  Biggs, J. and Tang, C. (2011).  Teaching for quality learning at university . Maidenhead: Open University Press.  Ramsay, P., Maier, P. and Price, G. (2010).  Study skills for business and management students . Harlow: Longman. 

Unless otherwise specified by your module leader, the University uses the Harvard (author-date) style of citing and referencing. For more guidance and support on how to reference effectively check out the  Referencing Guide . You can also  book an appointment  with an Academic Engagement Librarian for extra help with referencing. 

While the main results of your study should be placed in the body of your dissertation, any extra information can be placed in the appendices chapter. This supplementary information, for instance, can consist of graphs, charts, or tables that demonstrate less significant results or interview transcripts that would disrupt the flow of the main text if they were included within it.  

You can create one long appendix section or divide it into smaller sections to make it easier to navigate. For example, you might want to have an appendix for images, an appendix for transcripts, and an appendix for graphs. Each appendix (each graph or chart, etc.) should have its own number and title. Further, the sources for all appendices should be acknowledged through referencing and listed in the bibliography. 

Don’t forget to mention each appendix at least once during your dissertation! This can be done using brackets in the following way: (see appendix 1). 

  • << Previous: Standard
  • Last Updated: Nov 23, 2021 3:47 PM
  • URL: https://libguides.westminster.ac.uk/c.php?g=692395

CONNECT WITH US

  • FindAMasters
  • Researching and Writing a Masters Dissertation

Written by Mark Bennett

All Masters programmes include some form of extended individual project. Research-focussed programmes, such as an MRes , may include multiple independent research components. Taught courses usually culminate with a substantial research task, referred to as the Masters dissertation or thesis.

This article talks about how long a Masters dissertation is and the structure it follows.Before you get started on your dissertation, you'll usually need to write a proposal. Read our full guide to Masters dissertation proposals for more information on what this should include!

Masters dissertation - key facts
Length 15,000 - 20,000 words
Structure

Abstract (300 words)

Introduction (1,000 words)

Literature review (1,000 words)

Research methodology (1,500 words)

Results

Discussion (12,000 words)

Conclusion (1,500 words)

References/Bibliography

Appendices

Supervision Yes, you’ll be paired with an academic from your own university
Assessment External examiner along with additional members of faculty. There is not usually a viva at Masters level.

On this page

What’s the difference between a masters dissertation and an undergraduate dissertation.

The Masters thesis is a bridge between undergraduate study and higher level postgraduate degrees such as the PhD .

A postgraduate dissertation may not look that different to its undergraduate equivalent. You’ll likely have to produce a longer piece of work but the foundations remain the same.

After all, one of the purposes of an undergraduate dissertation or final year project is to prepare you for more in-depth research work as a postgraduate. That said, there are some important differences between the two levels.

So, how long is a Masters dissertation? A Masters dissertation will be longer than the undergraduate equivalent – usually it’ll be somewhere between 15,000 and 20,000 words, but this can vary widely between courses, institutions and countries.

To answer your overall research question comprehensively, you’ll be expected to identify and examine specific areas of your topic. This can be like producing a series of shorter pieces of work, similar to those required by individual modules. However, there’s the additional requirement that they collectively support a broader set of conclusions.

This more involved Masters dissertation structure will:

  • Give you the scope to investigate your subject in greater detail than is possible at undergraduate level
  • Challenge you to be effective at organising your work so that its individual components function as stages in a coherent and persuasive overall argument
  • Allow you to develop and hone a suitable research methodology (for example, choosing between qualitative and quantitative methods)

If the individual topics within your overall project require you to access separate sources or datasets, this may also have an impact on your research process.

As a postgraduate, you’ll be expected to establish and assert your own critical voice as a member of the academic community associated with your field .

During your Masters thesis you’ll need to show that you are not just capable of analysing and critiquing original data or primary source material. You should also demonstrate awareness of the existing body of scholarship relating to your topic .

So, if you’ll excuse the pun, a ‘Masters’ degree really is about achieving ‘mastery’ of your particular specialism and the dissertation is where you’ll demonstrate this: showing off the scholarly expertise and research skills that you’ve developed across your programme.

What’s the difference between a dissertation and a thesis?

A dissertation is a long piece of (usually) written work on the same topic. A thesis is a little more specific: it usually means something that presents an original argument based on the interpretation of data, statistics or content.

So, a thesis is almost always presented as a dissertation, but not all dissertations present a thesis.

Masters dissertation structure

As you can probably imagine, no two dissertations follow the exact same structure, especially given the differences found between Masters programmes from university to university and country to country .

That said, there are several key components that make up the structure of a typical Masters dissertation

How long is a Masters dissertation?

Most dissertations will typically be between 15,000 and 20,000 words long, although this can vary significantly depending on the nature of the programme.

You should also check with your university exactly which sections of the dissertation count towards the final word count (the abstract, bibliography and appendices won’t usually be included in the total).

Usually around 300 words long, the abstract is meant to be a concise summary of your dissertation. It should briefly cover the question(s) you aim to answer, your primary argument and your conclusion.

Introduction

The purpose of the introduction is to provide context for the rest of the dissertation, setting out your aims and the scope of what you want to achieve with your research. The introduction should give a clear overview of the dissertation’s chapters and will usually be around 1,000 words long.

Literature review

This part of the dissertation should examine the scholarship that has already been published in your field, presenting various arguments and counter-arguments while situating your own research within this wider body of work.

You should analyse and evaluate other publications and explain how your dissertation will contribute to the existing literature in your subject area. The literature review sometimes forms part of the introduction or follows immediately on from it. Most literature reviews are up to 1,000 words long.

Research methodology

Not all dissertations will require a section covering research methodology (Arts and Humanities dissertations won’t normally undertake the kind of research that involves a set methodology). However, if you are using a particular method to collect information for your dissertation, you should make sure to explain the rationale behind your choice of methodology. The word count for this part of the dissertation is usually around the 1,500 mark.

Those in the Arts and Humanities will usually outline their theoretical perspectives and approaches as part of the introduction, rather than requiring a detailed explanation of the methodology for their data collection and analysis.

Results / findings

If your research involves some form of survey or experiment, this is where you’ll present the results of your work. Depending on the nature of the study, this might be in the form of graphs, tables or charts – or even just a written description of what the research entailed and what the findings were.

This section forms the bulk of your dissertation and should be carefully structured using a series of related chapters (and sub-chapters). There should be a logical progression from one chapter to the next, with each part building on the arguments of its predecessor.

It can be helpful to think of your Masters dissertation as a series of closely interlinked essays, rather than one overwhelming paper. The size of this section will depend on the overall word count for your dissertation. However, to give you a rough idea for a 15,000-word dissertation, the discussion part will generally be about 12,000 words long.

Here you should draw together the threads of the previous discussion chapters and make your final concluding statements, drawing on evidence and arguments that you’ve already explored over the course of the dissertation. Explain the significance of your findings and point towards directions that future research could follow. This section of the Masters thesis will be around 1,500 words long.

References / bibliography

While planning and writing your dissertation, you should keep an extensive, organised record of any papers, sources or books you’ve quoted (or referred to). This will be a lot easier than leaving all of it until the end and struggling to work out where a particular quotation is from!

Appendices won’t be necessary in many dissertations, but you may need to include supplementary material to support your argument. This could be interview transcripts or questionnaires. If including such content within the body of the dissertation won’t be feasible – i.e. there wouldn’t be enough space or it would break the flow of your writing – you should consult with your supervisor and consider attaching it in an appendix.

It’s worth bearing in mind that these sections won’t always be discretely labelled in every dissertation. For example, everything up to ‘discussion’ might be covered in introductory chapter (rather than as distinct sections). If you’re unsure about the structure of your Masters dissertation, your supervisor will be able to help you map it out.

How does supervision work for a Masters dissertation?

As a Masters student at the dissertation stage you’ll usually be matched with an academic within your institution who will be tasked with guiding your work. This might be someone who has already taught you, or it may be another scholar whose research interests and expertise align well with what you want to do. You may be able to request a particular supervisor, but taught postgraduates are more likely to be assigned them by their department.

Specific arrangements with your supervisor will vary depending on your institution and subject area. They will usually meet with you at the beginning of the dissertation period to discuss your project and agree a suitable schedule for its undertaking. This timetable will probably set dates for:

  • Subsequent discussions and progress checks
  • The submission of draft chapters or sections
  • Feedback appointments

Though your supervisor is there to help and advise you, it is important to remember that your dissertation is a personal research project with associated expectations of you as an independent scholar.

As a rule of thumb, you can expect your supervisor to read each part of your dissertation once at the draft stage and to offer feedback. Most will not have time to look at lots of subsequent revisions, but may respond favourably to polite requests for exceptions (provided their own workload permits it).

Inundating your supervisor with emails or multiple iterations of draft material is best avoided; they will have their own research to manage (as well as other supervision assignments) and will be able to offer better quality feedback if you stick to an agreed schedule.

How is a Masters dissertation assessed and examined?

On most courses your dissertation will be assessed by an external examiner (as well as additional members of faculty within your university who haven’t been responsible for supervising you), but these will read and critique the work you submit without personally questioning and testing you on it.

Though this examination process is not as challenging as the oral defence or ‘ viva voce ’ required for a PhD thesis, the grading of your Masters dissertation is still a fundamental component of your degree.

On some programmes the result awarded to a student’s dissertation may determine the upper grade-band that can be awarded to their degree.

Search for a Masters

Ready to start looking for your ideal postgraduate opportunity? Browse and compare Masters degrees on FindAMasters.com.

Our postgrad newsletter shares courses, funding news, stories and advice

You may also like....

how long is a humanities dissertation

Applying for a Masters can feel a bit daunting. Here is a checklist of all the things you need to do to make sure you have everything covered in your Masters application.

how long is a humanities dissertation

Postgraduate study is often very flexible, with the option to study a Masters degree or other qualification part-time, online or through blended learning.

how long is a humanities dissertation

How do Bachelors and Masters courses differ? We’ve covered the main differences you’ll encounter when making the transition from undergrad to postgrad study.

how long is a humanities dissertation

Our guide explains how online Masters degree work, what the benefits of online learning are and how to choose what to study online.

how long is a humanities dissertation

Our guide tells you everything about the application process for studying a Masters in Italy.

how long is a humanities dissertation

Our guide tells you everything about the application process for studying a Masters in the USA.

FindAMasters. Copyright 2005-2024 All rights reserved.

Unknown    ( change )

Have you got time to answer some quick questions about Masters study?

Select your nearest city

  • Aberystwyth
  • Beaconsfield
  • Bishop Burton
  • Bournemouth
  • Bridlington
  • Chatham Maritime
  • Cirencester
  • East Malling
  • Hemel Hempstead
  • High Wycombe
  • Huddersfield
  • Isle of Man
  • Jordanstown
  • London Central
  • London East
  • London South
  • London West
  • Londonderry
  • Loughborough
  • Middlesbrough
  • Milton Keynes
  • Musselburgh
  • Northampton
  • Potters Bar
  • Saffron Waldon
  • Scarborough
  • Southampton
  • St Leonards on Sea
  • Stoke on Trent
  • Wolverhampton

You haven’t completed your profile yet. To get the most out of FindAMasters, finish your profile and receive these benefits:

  • Monthly chance to win one of ten £10 Amazon vouchers ; winners will be notified every month.*
  • Access to our £6,000 scholarship competition
  • Weekly newsletter with funding opportunities, application tips and much more
  • Early access to our physical and virtual postgraduate study fairs

Or begin browsing FindAMasters.com

or begin browsing FindAMasters.com

*Offer only available for the duration of your active subscription, and subject to change. You MUST claim your prize within 72 hours, if not we will redraw.

how long is a humanities dissertation

Do you want hassle-free information and advice?

Create your FindAMasters account and sign up to our newsletter:

  • Find out about funding opportunities and application tips
  • Receive weekly advice, student stories and the latest Masters news
  • Hear about our upcoming study fairs
  • Save your favourite courses, track enquiries and get personalised subject updates

how long is a humanities dissertation

Create your account

Looking to list your Masters courses? Log in here .

Modal image

Let us help you find a Masters

Never miss a course

Enter our ambassador competition

Get funding news, tips and advice

Hear about upcoming events

Sign up to our newsletter today

We've been helping students find the right postgraduate course for over a decade.

Login to your account

Enter your username below to login to your account.

The Writing Center • University of North Carolina at Chapel Hill

What this handout is about

This handout provides definitions and examples of the two main types of abstracts: descriptive and informative. It also provides guidelines for constructing an abstract and general tips for you to keep in mind when drafting. Finally, it includes a few examples of abstracts broken down into their component parts.

What is an abstract?

An abstract is a self-contained, short, and powerful statement that describes a larger work. Components vary according to discipline. An abstract of a social science or scientific work may contain the scope, purpose, results, and contents of the work. An abstract of a humanities work may contain the thesis, background, and conclusion of the larger work. An abstract is not a review, nor does it evaluate the work being abstracted. While it contains key words found in the larger work, the abstract is an original document rather than an excerpted passage.

Why write an abstract?

You may write an abstract for various reasons. The two most important are selection and indexing. Abstracts allow readers who may be interested in a longer work to quickly decide whether it is worth their time to read it. Also, many online databases use abstracts to index larger works. Therefore, abstracts should contain keywords and phrases that allow for easy searching.

Say you are beginning a research project on how Brazilian newspapers helped Brazil’s ultra-liberal president Luiz Ignácio da Silva wrest power from the traditional, conservative power base. A good first place to start your research is to search Dissertation Abstracts International for all dissertations that deal with the interaction between newspapers and politics. “Newspapers and politics” returned 569 hits. A more selective search of “newspapers and Brazil” returned 22 hits. That is still a fair number of dissertations. Titles can sometimes help winnow the field, but many titles are not very descriptive. For example, one dissertation is titled “Rhetoric and Riot in Rio de Janeiro.” It is unclear from the title what this dissertation has to do with newspapers in Brazil. One option would be to download or order the entire dissertation on the chance that it might speak specifically to the topic. A better option is to read the abstract. In this case, the abstract reveals the main focus of the dissertation:

This dissertation examines the role of newspaper editors in the political turmoil and strife that characterized late First Empire Rio de Janeiro (1827-1831). Newspaper editors and their journals helped change the political culture of late First Empire Rio de Janeiro by involving the people in the discussion of state. This change in political culture is apparent in Emperor Pedro I’s gradual loss of control over the mechanisms of power. As the newspapers became more numerous and powerful, the Emperor lost his legitimacy in the eyes of the people. To explore the role of the newspapers in the political events of the late First Empire, this dissertation analyzes all available newspapers published in Rio de Janeiro from 1827 to 1831. Newspapers and their editors were leading forces in the effort to remove power from the hands of the ruling elite and place it under the control of the people. In the process, newspapers helped change how politics operated in the constitutional monarchy of Brazil.

From this abstract you now know that although the dissertation has nothing to do with modern Brazilian politics, it does cover the role of newspapers in changing traditional mechanisms of power. After reading the abstract, you can make an informed judgment about whether the dissertation would be worthwhile to read.

Besides selection, the other main purpose of the abstract is for indexing. Most article databases in the online catalog of the library enable you to search abstracts. This allows for quick retrieval by users and limits the extraneous items recalled by a “full-text” search. However, for an abstract to be useful in an online retrieval system, it must incorporate the key terms that a potential researcher would use to search. For example, if you search Dissertation Abstracts International using the keywords “France” “revolution” and “politics,” the search engine would search through all the abstracts in the database that included those three words. Without an abstract, the search engine would be forced to search titles, which, as we have seen, may not be fruitful, or else search the full text. It’s likely that a lot more than 60 dissertations have been written with those three words somewhere in the body of the entire work. By incorporating keywords into the abstract, the author emphasizes the central topics of the work and gives prospective readers enough information to make an informed judgment about the applicability of the work.

When do people write abstracts?

  • when submitting articles to journals, especially online journals
  • when applying for research grants
  • when writing a book proposal
  • when completing the Ph.D. dissertation or M.A. thesis
  • when writing a proposal for a conference paper
  • when writing a proposal for a book chapter

Most often, the author of the entire work (or prospective work) writes the abstract. However, there are professional abstracting services that hire writers to draft abstracts of other people’s work. In a work with multiple authors, the first author usually writes the abstract. Undergraduates are sometimes asked to draft abstracts of books/articles for classmates who have not read the larger work.

Types of abstracts

There are two types of abstracts: descriptive and informative. They have different aims, so as a consequence they have different components and styles. There is also a third type called critical, but it is rarely used. If you want to find out more about writing a critique or a review of a work, see the UNC Writing Center handout on writing a literature review . If you are unsure which type of abstract you should write, ask your instructor (if the abstract is for a class) or read other abstracts in your field or in the journal where you are submitting your article.

Descriptive abstracts

A descriptive abstract indicates the type of information found in the work. It makes no judgments about the work, nor does it provide results or conclusions of the research. It does incorporate key words found in the text and may include the purpose, methods, and scope of the research. Essentially, the descriptive abstract describes the work being abstracted. Some people consider it an outline of the work, rather than a summary. Descriptive abstracts are usually very short—100 words or less.

Informative abstracts

The majority of abstracts are informative. While they still do not critique or evaluate a work, they do more than describe it. A good informative abstract acts as a surrogate for the work itself. That is, the writer presents and explains all the main arguments and the important results and evidence in the complete article/paper/book. An informative abstract includes the information that can be found in a descriptive abstract (purpose, methods, scope) but also includes the results and conclusions of the research and the recommendations of the author. The length varies according to discipline, but an informative abstract is rarely more than 10% of the length of the entire work. In the case of a longer work, it may be much less.

Here are examples of a descriptive and an informative abstract of this handout on abstracts . Descriptive abstract:

The two most common abstract types—descriptive and informative—are described and examples of each are provided.

Informative abstract:

Abstracts present the essential elements of a longer work in a short and powerful statement. The purpose of an abstract is to provide prospective readers the opportunity to judge the relevance of the longer work to their projects. Abstracts also include the key terms found in the longer work and the purpose and methods of the research. Authors abstract various longer works, including book proposals, dissertations, and online journal articles. There are two main types of abstracts: descriptive and informative. A descriptive abstract briefly describes the longer work, while an informative abstract presents all the main arguments and important results. This handout provides examples of various types of abstracts and instructions on how to construct one.

Which type should I use?

Your best bet in this case is to ask your instructor or refer to the instructions provided by the publisher. You can also make a guess based on the length allowed; i.e., 100-120 words = descriptive; 250+ words = informative.

How do I write an abstract?

The format of your abstract will depend on the work being abstracted. An abstract of a scientific research paper will contain elements not found in an abstract of a literature article, and vice versa. However, all abstracts share several mandatory components, and there are also some optional parts that you can decide to include or not. When preparing to draft your abstract, keep the following key process elements in mind:

  • Reason for writing: What is the importance of the research? Why would a reader be interested in the larger work?
  • Problem: What problem does this work attempt to solve? What is the scope of the project? What is the main argument/thesis/claim?
  • Methodology: An abstract of a scientific work may include specific models or approaches used in the larger study. Other abstracts may describe the types of evidence used in the research.
  • Results: Again, an abstract of a scientific work may include specific data that indicates the results of the project. Other abstracts may discuss the findings in a more general way.
  • Implications: What changes should be implemented as a result of the findings of the work? How does this work add to the body of knowledge on the topic?

(This list of elements is adapted with permission from Philip Koopman, “How to Write an Abstract.” )

All abstracts include:

  • A full citation of the source, preceding the abstract.
  • The most important information first.
  • The same type and style of language found in the original, including technical language.
  • Key words and phrases that quickly identify the content and focus of the work.
  • Clear, concise, and powerful language.

Abstracts may include:

  • The thesis of the work, usually in the first sentence.
  • Background information that places the work in the larger body of literature.
  • The same chronological structure as the original work.

How not to write an abstract:

  • Do not refer extensively to other works.
  • Do not add information not contained in the original work.
  • Do not define terms.

If you are abstracting your own writing

When abstracting your own work, it may be difficult to condense a piece of writing that you have agonized over for weeks (or months, or even years) into a 250-word statement. There are some tricks that you could use to make it easier, however.

Reverse outlining:

This technique is commonly used when you are having trouble organizing your own writing. The process involves writing down the main idea of each paragraph on a separate piece of paper– see our short video . For the purposes of writing an abstract, try grouping the main ideas of each section of the paper into a single sentence. Practice grouping ideas using webbing or color coding .

For a scientific paper, you may have sections titled Purpose, Methods, Results, and Discussion. Each one of these sections will be longer than one paragraph, but each is grouped around a central idea. Use reverse outlining to discover the central idea in each section and then distill these ideas into one statement.

Cut and paste:

To create a first draft of an abstract of your own work, you can read through the entire paper and cut and paste sentences that capture key passages. This technique is useful for social science research with findings that cannot be encapsulated by neat numbers or concrete results. A well-written humanities draft will have a clear and direct thesis statement and informative topic sentences for paragraphs or sections. Isolate these sentences in a separate document and work on revising them into a unified paragraph.

If you are abstracting someone else’s writing

When abstracting something you have not written, you cannot summarize key ideas just by cutting and pasting. Instead, you must determine what a prospective reader would want to know about the work. There are a few techniques that will help you in this process:

Identify key terms:

Search through the entire document for key terms that identify the purpose, scope, and methods of the work. Pay close attention to the Introduction (or Purpose) and the Conclusion (or Discussion). These sections should contain all the main ideas and key terms in the paper. When writing the abstract, be sure to incorporate the key terms.

Highlight key phrases and sentences:

Instead of cutting and pasting the actual words, try highlighting sentences or phrases that appear to be central to the work. Then, in a separate document, rewrite the sentences and phrases in your own words.

Don’t look back:

After reading the entire work, put it aside and write a paragraph about the work without referring to it. In the first draft, you may not remember all the key terms or the results, but you will remember what the main point of the work was. Remember not to include any information you did not get from the work being abstracted.

Revise, revise, revise

No matter what type of abstract you are writing, or whether you are abstracting your own work or someone else’s, the most important step in writing an abstract is to revise early and often. When revising, delete all extraneous words and incorporate meaningful and powerful words. The idea is to be as clear and complete as possible in the shortest possible amount of space. The Word Count feature of Microsoft Word can help you keep track of how long your abstract is and help you hit your target length.

Example 1: Humanities abstract

Kenneth Tait Andrews, “‘Freedom is a constant struggle’: The dynamics and consequences of the Mississippi Civil Rights Movement, 1960-1984” Ph.D. State University of New York at Stony Brook, 1997 DAI-A 59/02, p. 620, Aug 1998

This dissertation examines the impacts of social movements through a multi-layered study of the Mississippi Civil Rights Movement from its peak in the early 1960s through the early 1980s. By examining this historically important case, I clarify the process by which movements transform social structures and the constraints movements face when they try to do so. The time period studied includes the expansion of voting rights and gains in black political power, the desegregation of public schools and the emergence of white-flight academies, and the rise and fall of federal anti-poverty programs. I use two major research strategies: (1) a quantitative analysis of county-level data and (2) three case studies. Data have been collected from archives, interviews, newspapers, and published reports. This dissertation challenges the argument that movements are inconsequential. Some view federal agencies, courts, political parties, or economic elites as the agents driving institutional change, but typically these groups acted in response to the leverage brought to bear by the civil rights movement. The Mississippi movement attempted to forge independent structures for sustaining challenges to local inequities and injustices. By propelling change in an array of local institutions, movement infrastructures had an enduring legacy in Mississippi.

Now let’s break down this abstract into its component parts to see how the author has distilled his entire dissertation into a ~200 word abstract.

What the dissertation does This dissertation examines the impacts of social movements through a multi-layered study of the Mississippi Civil Rights Movement from its peak in the early 1960s through the early 1980s. By examining this historically important case, I clarify the process by which movements transform social structures and the constraints movements face when they try to do so.

How the dissertation does it The time period studied in this dissertation includes the expansion of voting rights and gains in black political power, the desegregation of public schools and the emergence of white-flight academies, and the rise and fall of federal anti-poverty programs. I use two major research strategies: (1) a quantitative analysis of county-level data and (2) three case studies.

What materials are used Data have been collected from archives, interviews, newspapers, and published reports.

Conclusion This dissertation challenges the argument that movements are inconsequential. Some view federal agencies, courts, political parties, or economic elites as the agents driving institutional change, but typically these groups acted in response to movement demands and the leverage brought to bear by the civil rights movement. The Mississippi movement attempted to forge independent structures for sustaining challenges to local inequities and injustices. By propelling change in an array of local institutions, movement infrastructures had an enduring legacy in Mississippi.

Keywords social movements Civil Rights Movement Mississippi voting rights desegregation

Example 2: Science Abstract

Luis Lehner, “Gravitational radiation from black hole spacetimes” Ph.D. University of Pittsburgh, 1998 DAI-B 59/06, p. 2797, Dec 1998

The problem of detecting gravitational radiation is receiving considerable attention with the construction of new detectors in the United States, Europe, and Japan. The theoretical modeling of the wave forms that would be produced in particular systems will expedite the search for and analysis of detected signals. The characteristic formulation of GR is implemented to obtain an algorithm capable of evolving black holes in 3D asymptotically flat spacetimes. Using compactification techniques, future null infinity is included in the evolved region, which enables the unambiguous calculation of the radiation produced by some compact source. A module to calculate the waveforms is constructed and included in the evolution algorithm. This code is shown to be second-order convergent and to handle highly non-linear spacetimes. In particular, we have shown that the code can handle spacetimes whose radiation is equivalent to a galaxy converting its whole mass into gravitational radiation in one second. We further use the characteristic formulation to treat the region close to the singularity in black hole spacetimes. The code carefully excises a region surrounding the singularity and accurately evolves generic black hole spacetimes with apparently unlimited stability.

This science abstract covers much of the same ground as the humanities one, but it asks slightly different questions.

Why do this study The problem of detecting gravitational radiation is receiving considerable attention with the construction of new detectors in the United States, Europe, and Japan. The theoretical modeling of the wave forms that would be produced in particular systems will expedite the search and analysis of the detected signals.

What the study does The characteristic formulation of GR is implemented to obtain an algorithm capable of evolving black holes in 3D asymptotically flat spacetimes. Using compactification techniques, future null infinity is included in the evolved region, which enables the unambiguous calculation of the radiation produced by some compact source. A module to calculate the waveforms is constructed and included in the evolution algorithm.

Results This code is shown to be second-order convergent and to handle highly non-linear spacetimes. In particular, we have shown that the code can handle spacetimes whose radiation is equivalent to a galaxy converting its whole mass into gravitational radiation in one second. We further use the characteristic formulation to treat the region close to the singularity in black hole spacetimes. The code carefully excises a region surrounding the singularity and accurately evolves generic black hole spacetimes with apparently unlimited stability.

Keywords gravitational radiation (GR) spacetimes black holes

Works consulted

We consulted these works while writing this handout. This is not a comprehensive list of resources on the handout’s topic, and we encourage you to do your own research to find additional publications. Please do not use this list as a model for the format of your own reference list, as it may not match the citation style you are using. For guidance on formatting citations, please see the UNC Libraries citation tutorial . We revise these tips periodically and welcome feedback.

Belcher, Wendy Laura. 2009. Writing Your Journal Article in Twelve Weeks: A Guide to Academic Publishing Success. Thousand Oaks, CA: Sage Press.

Koopman, Philip. 1997. “How to Write an Abstract.” Carnegie Mellon University. October 1997. http://users.ece.cmu.edu/~koopman/essays/abstract.html .

Lancaster, F.W. 2003. Indexing And Abstracting in Theory and Practice , 3rd ed. London: Facet Publishing.

You may reproduce it for non-commercial use if you use the entire handout and attribute the source: The Writing Center, University of North Carolina at Chapel Hill

Make a Gift

Search Website Search

Writing in the Humanities and Arts

Graduate Student Resource Center

Workshop Videos on Writing in the Humanities and Arts

These recorded versions of GWC workshops cover journal article writing and publishing, seminar papers, and conference presentations in the humanities.

Blanpain, Kristin. Academic Writing in the Humanities and Social Sciences: A Resource for Researchers. 2006. This workbook offers explanations, examples, and exercises designed to help scholars improve the grammar and flow of their writing. It includes discussions of academic style and academic genres (literature reviews, abstracts, research articles, etc.).

Hayot, Eric. The Elements of Academic Style: Writing for the Humanities . 2014. This book advises writers on granular concerns, such as sentence structure and grammar, as well as big-picture issues, such as adhering to genre patterns for successful research and publishing and developing productive and rewarding writing habits.

Dissertations and Theses

AHA Graduate and Early Career Committee. From Concept to Completion: A Dissertation-Writing Guide for History Students. 2008. Twelve historians offer advice on the dissertation-writing process in the field of history. Includes chapters on choosing a topic, obtaining funding, managing the dissertation committee, organizing archival materials, using sources, and overcoming writer’s block.

Bolker, Joan. Writing Your Dissertation in Fifteen Minutes a Day: A Guide to Starting, Revising, and Finishing Your Doctoral Thesis. 1998. This is one of several “write your dissertation” guides on the market, and it’s one of relatively few that gears itself toward writers of all disciplines. Bolker here is part career counselor, part writing coach, and part therapist. She seems particularly interested in the ways that graduate students block themselves from completing the dissertation through fear, ambivalence, procrastination, etc. Recommended as a general reference on the dissertation process, although some issues might require more specialized help for individual writers.

Clark, Irene L. Writing the Successful Thesis and Dissertation: Entering the Conversation. 2006. This title is regarded by many in composition studies as the best book on dissertation writing. Some of the writing strategies may be oriented more towards the humanities and social sciences, but the book offers excellent advice on writing process issues that is helpful to graduate students in all fields.

Lovitts, Barbara E. and Ellen Wert. Developing Quality Dissertations in the Humanities: A Graduate Student’s Guide to Achieving Excellence. 2008. Stylus Publishing. A concise booklet designed to define and explain expectations for dissertations in the humanities.

Zerubavel, Eviatar. The Clockwork Muse: A Practical Guide to Writing Theses, Dissertations, and Books. 1999. Working from the premise that "It is methodicalness and routinization. . . that help us produce theses, dissertations, and books," (3), this short book presents a detailed process for coming up with a realistic writing schedule and deadlines. Zerubavel explains management strategies for long writing projects: scheduling regular writing time, making outlines, setting realistic expectations, adhering to deadlines, etc.

This GWC-produced video workshop describes the process of preparing, submitting, and revising a humanities article for publication in a scholarly journal: Introduction to Publishing Journal Articles in the Humanities

Belcher, Wendy Laura. Writing Your Journal Article in Twelve Weeks . 2009. This workbook takes the approach to scheduling and work style that productivity scholars recommend and tailors its advice to the specificity of the journal article for humanities and social scientists. It is helpful on several fronts: first, in breaking down the process of writing the article into manageable parts and second, in explaining several other genres that graduate students need to be aware of, particularly the abstract.

UCLA Editor in Residence Program : UCLA Humanities Division has hosted a number of talks by editors of academic presses who have presented on how to prepare book proposals and book publications. See especially the video archive of these presentations , which for the most part require a UCLA login.

The Association of University Presses provides a resource page for prospective faculty authors , including a page that provides resources for finding a publisher . The web page on finding a publisher links to an annual subject area grid that indicates which publishers have strong interests in which fields and sub-fields. There are additional resources on university presses and advice about getting published.

Grant and Fellowship Proposals

Gillis, Christina M. Writing Proposals for ACLS Fellowship Competitions . A guide for fellowship applicants in all fields of the humanities, with advice on tailoring the proposal for its audience, creating an effective structure, explaining the project's significance, and positioning research in relation to previous work.

The National Endowment for Humanities offers fellowships and grants that are not usually open to graduate students. However, NEH resources are generally informative for learning how to write about proposed research in the humanities. If you would like to learn more about how to write successful proposals, this web page offers a webinar recording and sample narratives from awardees. This article offers additional advice.

Library Research Guides by Subject

These online guides offer discipline-specific links and tips for library research at UCLA. They include links to databases, catalogues, dictionaries, encyclopedias, electronic texts, and online archives.

To report a broken link, please email us at [email protected] .

California State University, Long Beach

  • Colleges & Degrees
  • Academic Calendar
  • International Education
  • Graduate Studies
  • Accreditation
  • Tuition and Fees
  • Parking & Maps
  • Careers with CSULB
  • Alumni Home
  • Alumni Volunteering
  • Alumni Giving

Campus Life

  • Centers & Organizations
  • Commencement
  • Student Life
  • Office of the President
  • Office of the Provost
  • Administration & Finance
  • Student Affairs
  • University Relations & Development
  • Information Technology
  • Beach Shops
  • Campus Directory
  • Enrollment Services
  • Financial Aid
  • Schedule of Classes
  • Student Records
  • 49er Foundation
  • Research Foundation

California State University Long Beach

1250 BELLFLOWER BOULEVARD LONG BEACH, CALIFORNIA 90840 562.985.4111

person at laptop working

Thesis and Culminating Project Information

Culminating Activities

sculpture image on LH 150

A main degree requirement on all students in graduate programs in the College of Liberal Arts is the completion of a culminating project, such as a thesis and oral defense thereof, a final comprehensive examination, a portfolio, or a non-thesis project.

Master's theses are original academic disquisitions on a problem or issue of disciplinary interest. Compositionally, they explain the significance of the undertaking, normally by stating the major assumptions and reviewing the relevant literature, and then elaborate on the methods of data collection and analysis of evidence, develop and critique the appropriate logical inferences and argumentation, and offer conclusions or recommendations for future work. The average length of the body of a Master's thesis is around 70 double-spaced pages. 

A non-thesis project is an artistic or technical endeavor that is appropriate for the fine and applied arts or to more professional or vocational fields. Examples could include an ethnographic film, a novel or body of creative writing, or a significantly original software program. Non-thesis projects should include a project report. The average length of the body of a project report is around 30 double-spaced pages. Non-thesis projects need not be submitted for college review.

Students may not enroll in thesis or exam coursework until advanced to candidacy . For further information about theses, course units, and committee membership, including chairpersons or directors, please review the university thesis requirements . For further information about comprehensive examinations, portfolios, or non-thesis projects, please review the university exam requirements. For information about enrollment in GS700B, please review AS policy 18–03 and contact the Director of Graduate Studies with any additional questions.

Oral Defense

By Title V §40510 , an oral defense of the thesis is required of those students who pursue the thesis option for their culminating activity. For students who pursue the non-thesis project option, an oral defense is recommended but not required.

Thesis Deadlines

All theses from the College of Liberal Arts must ultimately be approved by all thesis committee members and Professor Sarah Schrank, Director of Graduate Studies, before being submitted to the University's Thesis Office. Theses submitted to the college for review must be in their finalized form, with all copyediting and formatting completed. The theses and signature pages are to be sent to Professor Sarah Schrank, Director of Graduate Studies ( [email protected] ). The college deadlines for theses submissions can be found here:

SemesterCollege DeadlinesUniversity DeadlinesDate Your Thesis
Fall 2024October 21, 2024November 1, 2024December 2024
Winter 2025December 16, 2024January 3, 2025January 2025
Spring 2025March 24, 2025April 3, 2025May 2025
Summer 2025June 30, 2025July 11, 2025August 2025

CSULB Thesis & Dissertation Office

The University's Thesis & Dissertation Office is housed on the 5 th floor of the CSULB Library in Room 501. There are numerous resources to help graduate students with the composition of their theses. Most importantly, please consult the University's formatting guidelines , as it details all formatting rules and requirements, and includes protocols for electronic submission . Various templates have also been provided, including the template for the thesis signature page , as well as Department-specific style guides . The Thesis Office also offers consultations for students prior to submission.

California State University, Long Beach

A white megaphone in front of a light blue background.

  • E-mail the Help Desk
  • User's Guide for H-Announce

2025 Hay-Nicolay Dissertation Prize Announcement

The Abraham Lincoln Association (ALA) and the Abraham Lincoln Institute (ALI) invite nominations and submissions for the 2025 Hay-Nicolay Dissertation Prize, which includes a $1000 honorarium. 

Eligible doctoral dissertations must have been completed between 2022 and 2024. Preference will be given to Abraham Lincoln-focused dissertations, but studies that examine Lincoln’s history and political, cultural, and/or social legacy within a larger context (such as emancipation during the Civil War, Reconstruction, Lincoln administration policies involving indigenous peoples), will also be considered. Comparative studies (e.g., leadership studies, biographies) offering more than one chapter of Lincoln-related material will be considered, as well as studies of individuals traditionally associated with Abraham Lincoln. Submissions should offer new evidence or primary source materials, new interpretations, and/or a reevaluation of previously accepted conclusions.

Dissertations written in a language other than English between 2020 and 2024 are eligible, but must be translated and submitted in English.

Selection will be made by a joint ALA/ALI committee. The winner will be notified by January 2025, with the expectation of accepting in person (travel and lodging reimbursed) at either the ALA banquet in February 2025, in Springfield, IL, or the ALI symposium in March 2025, at Ford’s Theatre in Washington, D.C.

Dissertations must be submitted by email as a PDF to Dr. Dan Monroe ( [email protected] ) and Dr. Michelle Krowl ( [email protected] ).

Submissions must be received by 5:00pm Eastern Time on Saturday, October 5, 2024.

The information above is also available online at https://lincoln-institute.org/dissertation-awards/  

Michelle Krowl

Co-chair, Hay-Nicolay Dissertation Prize committee

FDM: Quality Screens A Success For This Long-Running Micro-Cap ETF

The Sunday Investor profile picture

  • FDM tracks the Dow Jones Select Microcap Index, selecting about 150 micro-cap stocks after applying profitability and value screens. Expenses are 0.60% and the ETF has $181 million in assets.
  • These screens have proved valuable over the long run. Since 2005, it has outperformed the more popular and better diversified IWC with less risk. I expect this trend to continue.
  • My analysis reveals FDM is superior on risk management, value, and quality, with a slight weakness on growth. Ultimately, it looks better prepared for multiple market conditions.
  • Micro-cap stocks are speculative, but there could be an opportunity coming up, depending on the results of the November elections. They might also be due for a value re-rating soon based on recent returns.
  • As a result, readers with a high tolerance for risk should put FDM on their watchlist in place of IWC. The former is a "hold", but the latter is certainly a "sell".

Rolling the dice

Investment Thesis

The iShares Micro-Cap ETF ( IWC ) might be more popular, but the First Trust Dow Jones Select MicroCap Index Fund ( NYSEARCA: FDM ) is the superior choice in this often overlooked category. As I will demonstrate, FDM's selections are better valued, less risky, and are more profitable, which should lead to higher long-term returns. To be sure, there are some major risks to consider, including the difficulty in predicting the timing of risk-on environments that typically favor smaller stocks. As such, I've only assigned FDM a "hold" rating, but keep this one on your radar if you have an appetite for risk , and I look forward to taking you through how the strategy works in more detail below.

FDM Overview

Strategy discussion.

FDM tracks the Dow Jones Select Microcap Index, which launched on June 27, 2005, exactly three months before FDM's inception date. The Index is float-adjusted market-cap-weighted, reconstitutes annually in August, and selects from an eligible universe defined as the bottom two deciles by market cap for all NYSE-listed stocks. Afterward, the universe is narrowed down by the following size and liquidity indicators:

  • Within the top 1,000 stocks by full market capitalization.
  • Within the top 1,000 stocks by three-month average value traded.
  • Within the top 1,500 stocks by one-month average value traded.

The remaining universe is ranked on five financial indicators, as follows:

  • Trailing P/E (highest 20% excluded)
  • Trailing P/S (highest 20% excluded)
  • Per share profit change for last quarter (lowest 20% excluded).
  • Operating profit margin (lowest 20% excluded)
  • Six-month total return (lowest 20% excluded).

Lastly, the process removes stocks with negative or missing P/E ratios or negative operating profit margins. You might be surprised at how many stocks these reasonable screens eliminate. While IWC has 1,457 equity holdings, FDM only holds 157. That's what the "select" part of the ETF's name signifies. It uses standard value and quality screens to improve the portfolio's financial health, and although this decreases diversification, it's necessary to avoid substantial underperformance when micro-caps fall out of favor.

Performance Analysis

These basic screens have historically been successful. Since October 2005, FDM has delivered a 281.06% total return (7.33% annualized) compared to 186.64% (5.72%) annualized for IWC. There's no need to adjust for fees, as both ETFs have the same 0.60% expense ratio. Simply put, FDM's Index has proved superior in the long run. However, its $181 million in assets under management is only a fraction of IWC's $882 million.

FDM vs. IWC vs. SPY Performance

Portfolio Visualizer

When you buy quality and value, you purchase downside protection, and we see that in FDM's superior risk-adjusted returns. FDM's Sharpe and Sortino Ratios are better than IWC's, outperforming by 7% in 2008, the last major recession. Some downside protection is prudent since the Sahm Rule was triggered earlier this month. This rule indicates the economy is in or likely to be in a recession soon when the unemployment rate rises 0.50% above its twelve-month low. It's a "rule" because it's coincided with every recession since 1960.

Real Time Sahm Rule Recession Indicator

St. Louis Federal Reserve

Returning to performance, FDM only substantially underperformed IWC once over its long history. As shown below, it lagged behind by 24.97% in 2020, which supports an argument that IWC is much more speculative. Still, FDM's median annual outperformance between 2006 and 2023 was 1.10%, beating IWC in 10/18 years. Even in years when it underperformed, it was only by 2.77% on average (excluding 2020), so investors are not missing out on much growth potential by taking the safer route with FDM.

Annualized Active Return: FDM vs. IWC

FDM Analysis

Sector allocations.

Financials comprise 28.94% of FDM, 22.00% of which are small regional banks with market caps between $126 and $907 million. It seems excessive, but considering they account for about 17% of IWC, it's something micro-cap ETF investors must accept.

FDM Sector Exposures

First Trust

FDM also overweights Industrials by about 9% but has about 20% less exposure to Health Care, accomplished primarily by avoiding unprofitable biotech stocks.

FDM Fundamentals By Sub-Industry

The following table highlights selected fundamental metrics for FDM's top 25 holdings, totaling 74.40% of the portfolio. Surprisingly, this isn't much more concentrated than IWC's 65.95% figure, which softens IWC's "superior diversification" argument. Based on my read of the methodology document, this was not by design, but it does alleviate some of my concerns about FDM being too reliant on only a handful of segments.

FDM Fundamental Analysis By Sub-Industry

The Sunday Investor

Here are three additional observations to consider:

1. FDM has a lower five-year beta (1.15 vs. 1.25), consistent with the lower volatility it's experienced historically. It's partially attributed to overweighting Regional Banks, which have a 0.84 five-year beta. However, it's more related to FDM underweighting biotech stocks by about 12%. To illustrate, consider how the Virtus LifeSci Biotech Clinical Trials ETF ( BBC ) experienced a 67.16% drawdown between March 2021 and October 2023. This statistic is relevant because nearly half (45%) of BBC's holdings are in IWC.

Regional Banks vs. Biotech Stocks - Performance Analysis

Therefore, even though I have reservations about Regional Banks, IWC's solution to overweight these speculative stocks is no better from a risk or return perspective. This table , which covers the period between January 1, 2015, and August 16, 2024, also highlights how the Invesco KBW Regional Banking ETF ( KBWR ) was the better performer by far, even after the crisis early last year.

2. FDM trades at 0.72x sales and 6.22x cash flow and has a sector-adjusted 5.96/10 value score, which puts it in the top six percentile among the 900+ U.S. Equity ETFs I track. Undoubtedly, it's a great value play, and I also come to the same conclusion by analyzing its annualized active return vs. the SPDR S&P 500 ETF ( SPY ). As shown below, FDM has lagged behind SPY by 15.08% YTD after missing by 13.44% in 2023. Arguably, micro-caps are due for a value re-rating.

Annualized Active Return: FDM vs. SPY

Another catalyst could be the election of Donald Trump. In 2016, expectations of deregulation and tax cuts provided the foundation for domestic micro-cap stocks to outperform. This time around, we should expect more of the same. According to the 2024 Republican Party Platform , "Cut Regulations" and "Make Trump Tax Cuts Permanent" are the party's top two economic commitments. In addition, the proposal of a 10% universal tariff on all U.S. imports could benefit U.S. companies initially, but it would eventually contribute to higher inflation. It's a delicate balance that's difficult to get right, but I believe there will be a temporary opportunity should Trump win in November.

3. FDM has a 5.18/10 sector-adjusted growth score backed by 3.60% and 5.48% one-year trailing and forward sales growth rates. These figures are lower than IWC's and, in my view, FDM's primary weakness. However, FDM's three-year historical growth rate is better (13.08% vs. 11.75%), mainly because of it overweighting commodity-linked sub-industries like Oil & Gas E&P and Steel and underweighting Biotechnology, whose sales were relatively low over the last three years (2.83% annualized). FDM is also a higher-quality fund, as evidenced by its 5.00/10 profit score. While the score obviously isn't great, it does reflect the Index's basic profitability screens that provide some stability in a challenging category.

Investment Recommendation

The micro-cap space is highly volatile and one I tend to avoid because of its unpredictability and my preference for high-quality companies. Still, micro-caps can outperform in risk-on environments, and I expect there could be a short-term opportunity soon. Catalysts include Donald Trump's potential re-election in November or even a simple value re-rating, given how micro-caps have lagged behind substantially over the last two years.

If this category interests you, I strongly prefer FDM over IWC. Although it's less popular, consider that FDM:

  • has an identical 0.60% expense ratio.
  • has a lower five-year beta (1.15 vs. 1.25) based on current holdings.
  • has a superior long-term track record on both return and risk.
  • is more attractively valued based on sales and cash flow.
  • promotes quality through screens for profit change and operating margins.

IWC proponents might argue it's better diversified, and although that's technically accurate, it's only accomplished through overweighting 150+ non-profitable biotech stocks. In my view, that defines "diworsification", and the recent returns of BBC suggest it's not a sector you want to overweight. Therefore, I've assigned a neutral "hold" rating to FDM, and although IWC wasn't the focus of today's article, my rating certainly would be a "sell." Thank you for reading, and I look forward to your comments below.

This article was written by

The Sunday Investor profile picture

Analyst’s Disclosure: I/we have a beneficial long position in the shares of SPY either through stock ownership, options, or other derivatives. I wrote this article myself, and it expresses my own opinions. I am not receiving compensation for it (other than from Seeking Alpha). I have no business relationship with any company whose stock is mentioned in this article.

Seeking Alpha's Disclosure: Past performance is no guarantee of future results. No recommendation or advice is being given as to whether any investment is suitable for a particular investor. Any views or opinions expressed above may not reflect those of Seeking Alpha as a whole. Seeking Alpha is not a licensed securities dealer, broker or US investment adviser or investment bank. Our analysts are third party authors that include both professional investors and individual investors who may not be licensed or certified by any institute or regulatory body.

Recommended For You

About fdm etf.

SymbolLast Price% Chg

More on FDM

Related stocks.

SymbolLast Price% Chg
FDM--

Trending Analysis

Trending news.

how long is a humanities dissertation

Frequently asked questions

How long is a dissertation.

Dissertation word counts vary widely across different fields, institutions, and levels of education:

  • An undergraduate dissertation is typically 8,000–15,000 words
  • A master’s dissertation is typically 12,000–50,000 words
  • A PhD thesis is typically book-length: 70,000–100,000 words

However, none of these are strict guidelines – your word count may be lower or higher than the numbers stated here. Always check the guidelines provided by your university to determine how long your own dissertation should be.

Frequently asked questions: Dissertation

A dissertation prospectus or proposal describes what or who you plan to research for your dissertation. It delves into why, when, where, and how you will do your research, as well as helps you choose a type of research to pursue. You should also determine whether you plan to pursue qualitative or quantitative methods and what your research design will look like.

It should outline all of the decisions you have taken about your project, from your dissertation topic to your hypotheses and research objectives , ready to be approved by your supervisor or committee.

Note that some departments require a defense component, where you present your prospectus to your committee orally.

A thesis is typically written by students finishing up a bachelor’s or Master’s degree. Some educational institutions, particularly in the liberal arts, have mandatory theses, but they are often not mandatory to graduate from bachelor’s degrees. It is more common for a thesis to be a graduation requirement from a Master’s degree.

Even if not mandatory, you may want to consider writing a thesis if you:

  • Plan to attend graduate school soon
  • Have a particular topic you’d like to study more in-depth
  • Are considering a career in research
  • Would like a capstone experience to tie up your academic experience

The conclusion of your thesis or dissertation should include the following:

  • A restatement of your research question
  • A summary of your key arguments and/or results
  • A short discussion of the implications of your research

The conclusion of your thesis or dissertation shouldn’t take up more than 5–7% of your overall word count.

For a stronger dissertation conclusion , avoid including:

  • Important evidence or analysis that wasn’t mentioned in the discussion section and results section
  • Generic concluding phrases (e.g. “In conclusion …”)
  • Weak statements that undermine your argument (e.g., “There are good points on both sides of this issue.”)

Your conclusion should leave the reader with a strong, decisive impression of your work.

While it may be tempting to present new arguments or evidence in your thesis or disseration conclusion , especially if you have a particularly striking argument you’d like to finish your analysis with, you shouldn’t. Theses and dissertations follow a more formal structure than this.

All your findings and arguments should be presented in the body of the text (more specifically in the discussion section and results section .) The conclusion is meant to summarize and reflect on the evidence and arguments you have already presented, not introduce new ones.

A theoretical framework can sometimes be integrated into a  literature review chapter , but it can also be included as its own chapter or section in your dissertation . As a rule of thumb, if your research involves dealing with a lot of complex theories, it’s a good idea to include a separate theoretical framework chapter.

A literature review and a theoretical framework are not the same thing and cannot be used interchangeably. While a theoretical framework describes the theoretical underpinnings of your work, a literature review critically evaluates existing research relating to your topic. You’ll likely need both in your dissertation .

While a theoretical framework describes the theoretical underpinnings of your work based on existing research, a conceptual framework allows you to draw your own conclusions, mapping out the variables you may use in your study and the interplay between them.

A thesis or dissertation outline is one of the most critical first steps in your writing process. It helps you to lay out and organize your ideas and can provide you with a roadmap for deciding what kind of research you’d like to undertake.

Generally, an outline contains information on the different sections included in your thesis or dissertation , such as:

  • Your anticipated title
  • Your abstract
  • Your chapters (sometimes subdivided into further topics like literature review , research methods , avenues for future research, etc.)

When you mention different chapters within your text, it’s considered best to use Roman numerals for most citation styles. However, the most important thing here is to remain consistent whenever using numbers in your dissertation .

In most styles, the title page is used purely to provide information and doesn’t include any images. Ask your supervisor if you are allowed to include an image on the title page before doing so. If you do decide to include one, make sure to check whether you need permission from the creator of the image.

Include a note directly beneath the image acknowledging where it comes from, beginning with the word “ Note .” (italicized and followed by a period). Include a citation and copyright attribution . Don’t title, number, or label the image as a figure , since it doesn’t appear in your main text.

Definitional terms often fall into the category of common knowledge , meaning that they don’t necessarily have to be cited. This guidance can apply to your thesis or dissertation glossary as well.

However, if you’d prefer to cite your sources , you can follow guidance for citing dictionary entries in MLA or APA style for your glossary.

A glossary is a collection of words pertaining to a specific topic. In your thesis or dissertation, it’s a list of all terms you used that may not immediately be obvious to your reader. In contrast, an index is a list of the contents of your work organized by page number.

The title page of your thesis or dissertation goes first, before all other content or lists that you may choose to include.

The title page of your thesis or dissertation should include your name, department, institution, degree program, and submission date.

Glossaries are not mandatory, but if you use a lot of technical or field-specific terms, it may improve readability to add one to your thesis or dissertation. Your educational institution may also require them, so be sure to check their specific guidelines.

A glossary or “glossary of terms” is a collection of words pertaining to a specific topic. In your thesis or dissertation, it’s a list of all terms you used that may not immediately be obvious to your reader. Your glossary only needs to include terms that your reader may not be familiar with, and is intended to enhance their understanding of your work.

A glossary is a collection of words pertaining to a specific topic. In your thesis or dissertation, it’s a list of all terms you used that may not immediately be obvious to your reader. In contrast, dictionaries are more general collections of words.

An abbreviation is a shortened version of an existing word, such as Dr. for Doctor. In contrast, an acronym uses the first letter of each word to create a wholly new word, such as UNESCO (an acronym for the United Nations Educational, Scientific and Cultural Organization).

As a rule of thumb, write the explanation in full the first time you use an acronym or abbreviation. You can then proceed with the shortened version. However, if the abbreviation is very common (like PC, USA, or DNA), then you can use the abbreviated version from the get-go.

Be sure to add each abbreviation in your list of abbreviations !

If you only used a few abbreviations in your thesis or dissertation , you don’t necessarily need to include a list of abbreviations .

If your abbreviations are numerous, or if you think they won’t be known to your audience, it’s never a bad idea to add one. They can also improve readability, minimizing confusion about abbreviations unfamiliar to your reader.

A list of abbreviations is a list of all the abbreviations that you used in your thesis or dissertation. It should appear at the beginning of your document, with items in alphabetical order, just after your table of contents .

Your list of tables and figures should go directly after your table of contents in your thesis or dissertation.

Lists of figures and tables are often not required, and aren’t particularly common. They specifically aren’t required for APA-Style, though you should be careful to follow their other guidelines for figures and tables .

If you have many figures and tables in your thesis or dissertation, include one may help you stay organized. Your educational institution may require them, so be sure to check their guidelines.

A list of figures and tables compiles all of the figures and tables that you used in your thesis or dissertation and displays them with the page number where they can be found.

The table of contents in a thesis or dissertation always goes between your abstract and your introduction .

You may acknowledge God in your dissertation acknowledgements , but be sure to follow academic convention by also thanking the members of academia, as well as family, colleagues, and friends who helped you.

A literature review is a survey of credible sources on a topic, often used in dissertations , theses, and research papers . Literature reviews give an overview of knowledge on a subject, helping you identify relevant theories and methods, as well as gaps in existing research. Literature reviews are set up similarly to other  academic texts , with an introduction , a main body, and a conclusion .

An  annotated bibliography is a list of  source references that has a short description (called an annotation ) for each of the sources. It is often assigned as part of the research process for a  paper .  

In a thesis or dissertation, the discussion is an in-depth exploration of the results, going into detail about the meaning of your findings and citing relevant sources to put them in context.

The conclusion is more shorter and more general: it concisely answers your main research question and makes recommendations based on your overall findings.

In the discussion , you explore the meaning and relevance of your research results , explaining how they fit with existing research and theory. Discuss:

  • Your  interpretations : what do the results tell us?
  • The  implications : why do the results matter?
  • The  limitation s : what can’t the results tell us?

The results chapter or section simply and objectively reports what you found, without speculating on why you found these results. The discussion interprets the meaning of the results, puts them in context, and explains why they matter.

In qualitative research , results and discussion are sometimes combined. But in quantitative research , it’s considered important to separate the objective results from your interpretation of them.

Results are usually written in the past tense , because they are describing the outcome of completed actions.

The results chapter of a thesis or dissertation presents your research results concisely and objectively.

In quantitative research , for each question or hypothesis , state:

  • The type of analysis used
  • Relevant results in the form of descriptive and inferential statistics
  • Whether or not the alternative hypothesis was supported

In qualitative research , for each question or theme, describe:

  • Recurring patterns
  • Significant or representative individual responses
  • Relevant quotations from the data

Don’t interpret or speculate in the results chapter.

To automatically insert a table of contents in Microsoft Word, follow these steps:

  • Apply heading styles throughout the document.
  • In the references section in the ribbon, locate the Table of Contents group.
  • Click the arrow next to the Table of Contents icon and select Custom Table of Contents.
  • Select which levels of headings you would like to include in the table of contents.

Make sure to update your table of contents if you move text or change headings. To update, simply right click and select Update Field.

All level 1 and 2 headings should be included in your table of contents . That means the titles of your chapters and the main sections within them.

The contents should also include all appendices and the lists of tables and figures, if applicable, as well as your reference list .

Do not include the acknowledgements or abstract in the table of contents.

The abstract appears on its own page in the thesis or dissertation , after the title page and acknowledgements but before the table of contents .

An abstract for a thesis or dissertation is usually around 200–300 words. There’s often a strict word limit, so make sure to check your university’s requirements.

In a thesis or dissertation, the acknowledgements should usually be no longer than one page. There is no minimum length.

The acknowledgements are generally included at the very beginning of your thesis , directly after the title page and before the abstract .

Yes, it’s important to thank your supervisor(s) in the acknowledgements section of your thesis or dissertation .

Even if you feel your supervisor did not contribute greatly to the final product, you must acknowledge them, if only for a very brief thank you. If you do not include your supervisor, it may be seen as a snub.

In the acknowledgements of your thesis or dissertation, you should first thank those who helped you academically or professionally, such as your supervisor, funders, and other academics.

Then you can include personal thanks to friends, family members, or anyone else who supported you during the process.

Ask our team

Want to contact us directly? No problem.  We  are always here for you.

Support team - Nina

Our team helps students graduate by offering:

  • A world-class citation generator
  • Plagiarism Checker software powered by Turnitin
  • Innovative Citation Checker software
  • Professional proofreading services
  • Over 300 helpful articles about academic writing, citing sources, plagiarism, and more

Scribbr specializes in editing study-related documents . We proofread:

  • PhD dissertations
  • Research proposals
  • Personal statements
  • Admission essays
  • Motivation letters
  • Reflection papers
  • Journal articles
  • Capstone projects

Scribbr’s Plagiarism Checker is powered by elements of Turnitin’s Similarity Checker , namely the plagiarism detection software and the Internet Archive and Premium Scholarly Publications content databases .

The add-on AI detector is powered by Scribbr’s proprietary software.

The Scribbr Citation Generator is developed using the open-source Citation Style Language (CSL) project and Frank Bennett’s citeproc-js . It’s the same technology used by dozens of other popular citation tools, including Mendeley and Zotero.

You can find all the citation styles and locales used in the Scribbr Citation Generator in our publicly accessible repository on Github .

Advertisement

Supported by

The Australian Professor Who Turned Breaking on Its Head

Rachael Gunn, known as B-girl Raygun, displayed some … unique moves as she competed in a field with breakers half her age. The judges and the internet were underwhelmed.

  • Share full article

A woman wearing green track pants, a green polo shirt and a cap poses with her hand up in front of a judges table.

By Dodai Stewart and Talya Minsberg

Reporting from Paris

Breaking made its debut as an Olympic sport Friday, and among the competitors was Dr. Rachael Gunn, also known as B-girl Raygun, a 36-year-old professor from Sydney, Australia, who stood out in just about every way.

By day, her research interests include “dance, gender politics, and the dynamics between theoretical and practical methodologies.” But on the world’s stage in Paris, wearing green track pants and a green polo shirt instead of the street-style outfits of her much younger fellow breakers, she competed against the 21-year-old Logan Edra of the United States, known as Logistx.

During the round robin, as Raygun and Logistx faced off, Raygun laid on her side, reached for her toes, spun around, and threw in a kangaroo hop — a nod to her homeland. She performed a move that looked something like swimming and another that could best be described as duckwalking. The high-speed back and head spins that other breakers would demonstrate were mostly absent.

The crowd cheered Raygun politely. The judges weren’t as kind. All nine voted for Logistx in both rounds of the competition; Logistx won, 18-0.

Online, Raygun’s performance quickly became a sensation, not necessarily in a flattering way.

“The more I watch the videos of Raygun, the Aussie breaker, the more I get annoyed,” one viewer posted on X, formerly known as Twitter. “There’s 27.7 million Australians in the world and that’s who they send to the Olympics for this inaugural event??? C’mon now!”

We are having trouble retrieving the article content.

Please enable JavaScript in your browser settings.

Thank you for your patience while we verify access. If you are in Reader mode please exit and  log into  your Times account, or  subscribe  for all of The Times.

Thank you for your patience while we verify access.

Already a subscriber?  Log in .

Want all of The Times?  Subscribe .

IMAGES

  1. How Long Is a Dissertation?

    how long is a humanities dissertation

  2. A Detailed Overview Of How Long Is A Dissertation?

    how long is a humanities dissertation

  3. A Detailed Overview Of How Long Is A Dissertation?

    how long is a humanities dissertation

  4. How Long Is A Dissertation

    how long is a humanities dissertation

  5. How Long Is A Dissertation

    how long is a humanities dissertation

  6. How Long Is A Dissertation? A Guide To Planning Your Writing

    how long is a humanities dissertation

COMMENTS

  1. thesis

    The statement comes from a 2010 blog post named "How to write a doctoral dissertation in philosophy": "An American dissertation in Philosophy should be about 200 to 250 pages long, and have five or six chapters." Note that Wolff is talking about Philosophy specifically, not about other humanities as it is indicated in the question title.

  2. How to Write a Social Science or Humanities Thesis/Dissertation

    Humanities thesis/dissertations, on the other hand, are often structured more like long essays. This is because these theses/dissertations rely more heavily on discussions of previous literature and/or case studies. ... How long is a typical humanities thesis/dissertation? + A typical humanities thesis/dissertation tends to range from 100 to ...

  3. Tips for writing a PhD dissertation: FAQs answered

    A PhD thesis (or dissertation) is typically 60,000 to 120,000 words ( 100 to 300 pages in length) organised into chapters, divisions and subdivisions (with roughly 10,000 words per chapter) - from introduction (with clear aims and objectives) to conclusion. The structure of a dissertation will vary depending on discipline (humanities, social ...

  4. Writing a Dissertation: A Complete Guide

    How long should a dissertation introduction be? The unofficial rule is 10 percent of the entire paper, so if your dissertation is 20,000 words, your introduction should be about 2,000 words. Keep in mind this is a rough estimate, as your introduction could vary. Literature review.

  5. How long did you take your write your dissertation? : r/PhD

    In the humanities, this is measured in years, not months. It is standard in my field (Religious Studies) for the dissertation to take from 1.5 to 3 years. Humanities programs are typically in the 5-7 year expectation for the whole program, where the first 3-4 are coursework, exams, and prospectus. Once you're done with all that, you're ABD and ...

  6. Formatting Your Dissertation

    Thesis Acceptance Certificate: A copy of the Thesis Acceptance Certificate (TAC) should appear as the first page. This page should not be counted or numbered. The TAC will appear in the online version of the published dissertation. ... and—depending on the license—adapt the work so long as proper attribution is given. (If a student chooses ...

  7. PDF Humanities Dissertation Prospectus

    A prospectus should be viewed as a preliminary statement of what you propose to do in your dissertation, and not as an unalterable commitment. Its value lies in helping you and your committee get an initial handle on your project. In the Humanities Ph.D. Program, we have chosen to break the writing of the prospectus into 2 blocks:

  8. Dissertation and Defense

    A critical step in the progress towards the degree for doctoral students is the formation of the dissertation committee. Students should plan ahead in forming and maintaining their dissertation committee. ... as long as the dissertation committee adheres to the dissertation committee requirements stated above. ... Division of the Humanities The ...

  9. How Long Is a PhD Thesis?

    Unfortunately, there's no one size fits all answer to this question. However, from the analysis of over 100 PhD theses, the average thesis length is between 80,000 and 100,000 words. A further analysis of 1000 PhD thesis shows the average number of pages to be 204. In reality, the actual word count for each PhD thesis will depend on the ...

  10. PDF WRITING A DISSERTATION PROPOSAL IN THE HUMANITIES: HISTORY Note

    WRITING A DISSERTATION PROPOSAL IN THE HUMANITIES: HISTORY. Note: This information is intended to serve as a general guide to outlining a dissertation proposal in History. Specific expectations may vary by committee. You should always ascertain the requirements of your specific advisor/committee members before completing the dissertation proposal.

  11. What Is a Dissertation?

    A master's dissertation is typically 12,000-50,000 words; A PhD thesis is typically book-length: 70,000-100,000 words; However, none of these are strict guidelines - your word count may be lower or higher than the numbers stated here. Always check the guidelines provided by your university to determine how long your own dissertation ...

  12. Dissertations 2: Structure: Thematic

    In the humanities, a thematic dissertation is often structured like a long essay. It can contain: Title page. Abstract. Table of contents. Introduction. Literature review (which can be included in the introduction rather than as a separate chapter. Check with your supervisor if you are unsure). Theme 1.

  13. What Is a Dissertation?

    A dissertation is a long-form piece of academic writing based on original research conducted by you. It is usually submitted as the final step in order to finish a PhD program. ... and approach. Dissertations in the humanities are often structured more like a long essay, building an overall argument to support a central thesis, with chapters ...

  14. Writing the Humanities Dissertation Prospectus

    Humanities Workshops. Eric Newman, English, Graduate Writing Center Consultant. This workshop will focus on how to plan and begin writing your dissertation prospectus in the Humanities. It covers prospectus components, time and file management tips, and preparing for the defense. (Approx. 45 mins.) This workshop was recorded in 2018.

  15. PDF Guidelines for Undergraduate dissertations Faculty of Humanities

    Suggested approach: a 40 credit dissertation should be of 10,000-12,000 words and. 20 credit dissertation should be of 8,000-10,000 words. Allocation of supervisors. It is assumed that in most areas the student's preference for a supervisor will not distribute equally across the potential supervisors.

  16. Your Guide to Writing a Successful Masters Dissertation

    It can be helpful to think of your Masters dissertation as a series of closely interlinked essays, rather than one overwhelming paper. The size of this section will depend on the overall word count for your dissertation. However, to give you a rough idea for a 15,000-word dissertation, the discussion part will generally be about 12,000 words long.

  17. The Writing Center

    The conclusion leaves the reader with the information and/or impact that the writer wants; it is often what the reader remembers most by providing the final discharge of energy that the paper has built up. It is the writer's last chance to convince the reader. A conclusion often suggests larger implications now that the evidence has been ...

  18. Honors Theses

    In the humanities, theses average 50-75 pages in length and consist of two or more chapters. In the social sciences, the manuscript may be shorter, depending on whether the project involves more quantitative than qualitative research. ... you've spent a very long time on your thesis. Don't let minor blemishes (misspellings and incorrect ...

  19. Abstracts

    The Word Count feature of Microsoft Word can help you keep track of how long your abstract is and help you hit your target length. Example 1: Humanities abstract Kenneth Tait Andrews, "'Freedom is a constant struggle': The dynamics and consequences of the Mississippi Civil Rights Movement, 1960-1984" Ph.D. State University of New York ...

  20. What Is a Thesis?

    A thesis statement is a very common component of an essay, particularly in the humanities. It usually comprises 1 or 2 sentences in the introduction of your essay, and should clearly and concisely summarize the central points of your academic essay. A thesis is a long-form piece of academic writing, often taking more than a full semester to ...

  21. Writing in the Humanities and Arts

    From Concept to Completion: A Dissertation-Writing Guide for History Students. 2008. Twelve historians offer advice on the dissertation-writing process in the field of history. Includes chapters on choosing a topic, obtaining funding, managing the dissertation committee, organizing archival materials, using sources, and overcoming writer's block.

  22. How to Write a Thesis or Dissertation Conclusion

    A humanities dissertation topic or systematic review, on the other hand, might require more space to conclude its analysis, tying all the previous sections together in an overall argument. Step 1: Answer your research question. Your conclusion should begin with the main question that your thesis or dissertation aimed to address. This is your ...

  23. CLA Thesis and Culminating Project Information

    LONG BEACH, CALIFORNIA 90840 562.985.4111. Open Menu . College of Liberal Arts. CLA Home; About the College. Dean's Message; Administration and Staff; ... Thesis Deadlines. All theses from the College of Liberal Arts must ultimately be approved by all thesis committee members and Professor Sarah Schrank, Director of Graduate Studies, before ...

  24. 2025 Hay-Nicolay Dissertation Prize Announcement

    The Abraham Lincoln Association (ALA) and the Abraham Lincoln Institute (ALI) invite nominations and submissions for the 2025 Hay-Nicolay Dissertation Prize, which includes a $1000 honorarium. Eligible doctoral dissertations must have been completed between 2022 and 2024.

  25. FDM: Quality Screens A Success For This Long-Running Micro-Cap ETF

    Simply put, FDM's Index has proved superior in the long run. However, its $181 million in assets under management is only a fraction of IWC's $882 million. Portfolio Visualizer

  26. How long is a dissertation?

    How long is a dissertation? Dissertation word counts vary widely across different fields, institutions, and levels of education: An undergraduate dissertation is typically 8,000-15,000 words. A master's dissertation is typically 12,000-50,000 words. A PhD thesis is typically book-length: 70,000-100,000 words.

  27. How Raygun made it to the Olympics and divided breaking world

    Gunn was always a dancer - albeit in jazz, tap and ballroom first - but it was her husband and coach Samuel Free that introduced her to the world of breaking when she was 20.

  28. The Australian Professor Who Turned Breaking on Its Head

    Breaking made its debut as an Olympic sport Friday, and among the competitors was Dr. Rachael Gunn, also known as B-girl Raygun, a 36-year-old professor from Sydney, Australia, who stood out in ...